You are on page 1of 76

Mukho

Practice Single Best Answer Questions MRCOG Part II


Chapter 1
Antenatal and Labour complications

1. A G2 P1 lady presents with significant APH at 32 weeks of gestation. You take a detailed
history and diagnose a placental abruption.
What is the most important predictor for the risk of abruption?
A. History of maternal trauma following domestic violence
B. History of abruption in a previous pregnancy
C. History of maternal smoking
D. History of previous dilatation and curettage
E. History of maternal drug intake like cocaine

Answer: B
A number of clinical and epidemiological studies have identified predisposing risk factors for
placental abruption. The most predictive is abruption in a previous pregnancy. A large
observational study from Norway reported a 4.4% incidence of recurrent abruption (adjusted OR
7.8, 95% CI 6.5–9.2).
Abruption recurs in 19–25% of women who have had two previous pregnancies complicated by
abruption.
Other risk factors for placental abruption include:
- pre-eclampsia,
- fetal growth restriction,
- non-vertex presentations,
- polyhydramnios,
- advanced maternal age,
- multiparity,
- low body mass index (BMI),
- pregnancy following assisted reproductive techniques,
- intrauterine infection, premature rupture of membranes,
- abdominal trauma (both accidental and resulting from domestic violence),
- smoking and drug misuse (cocaine and amphetamines) during pregnancy.

2. A 22 year old unbooked primigravida presents to the A&E at 26 weeks of gestation with
a history of spontaneous painless bleeding of about 500 ml.
What is the best investigation to clinch a diagnosis?
A. MRI scan
B. Transabdominal scan
C. CTG
D. Transvaginal scan
E. CT scan
Answer: B
You should suspect a placenta praevia with a significant painless bleeding in a woman who is
unbooked (no anomaly scan to determine the position of the placenta). Hence, a transabdominal
ultrasound would be most appropriate to rule out placenta praevia and also establish fetal
wellbeing. GTG- Placenta praevia, Placenta Praevia Accreta and Vasa Praevia:

3. What is the incidence of velamentous insertion of umbilical cord into the placenta in an
unselected population?
A. 1:10,000
B. 1:100
C. 8:100
D. 8:1000
E. 1:1000

Answer: B

4. Women with one or more previous caesarean section scars and an anterior placenta are at
risk of placenta accrete.
Which test has been shown in recent research to provide the highest sensitivity and
specificity for antenatal diagnosis of placenta accrete?
A. Colour Doppler
B. 3-D power Doppler
C. Contrast CT
D. Gadolinium contrast MRI
E. Grey scale ultrasound

Answer: B

5. What is the fetal blood volume at term?


A. 10-20 ml/kg
B. 40-60 ml/kg
C. 80-100 ml/kg
D. 120-140 ml/kg
E. 150-170 ml/kg

Answer: C
The fetus has about 80-100 ml/kg blood volume at term hence when bleeding from a vasa previa,
the fetus can get exsanguinated very quickly; hence a diagnosis of vasa praevia should instigate a
category 1 caesarean section.
6. Approximately what percentage of women in the UK are offered invasive prenatal
diagnostic tests per annum?
A. 1%
B. 2%
C. 3%
D. 4%
E. 5%

Answer: E
Approximtaely 30,000 women in the UK are offered invasive tests such as amniocentesisi and
CVS. This constitutes 5% of the pregnant population.

7. Getting blood in the amniotic fluid during ultrasound-guided amniocentesis is a


recognized complication.
What is the incidence of this risk?
A. 1:1000
B. 5:1000
C. 10:1000
D. 25:1000
E. 50:1000

Answer: D? B
The best evidence of risks associated with amniocentesis comes from a randomized trial form
Danmark reported in 1986.
Despite ultrasound-guidance, a trans-placental approach may be required in around 10-15%
cases, some of which lead to a bloody tap. The risk of blood stained amniotic tap is 0.5%. The
procedure related pregnancy loss or miscarriage is around 1%.

8. What is the maximum size of needle recommended for use during amniocentesis at 15
weeks?
A. 14G
B. 16B
C. 18G
D. 20G
E. 26G

Answer: D
A 20 G needle is the maximum recommended size for minimal trauma while allowing optimum
aspiration of amniotic fluid.

9. A triple test is performed for Down’s screening at 16 weeks in a 40-year-old woman. The
results suggest a high risk of trisomy 21.
What would the results typically show?
A. Reduced AFP, reduced estriol, increased β-hCG
B. Increased AFP, reduced estriol, increased β-hCG
C. Reduced AFP, increased estriol, increased β-hCG
D. Reduced AFP, increased estriol, reduced β-hCG
E. Increased AFP, increased estriol, increased β-hCG

Answer: A
The maternal serum alpha feto-protein and unconjugated estriol both show a reduction by about
25% of the normal and the β-hCG may increase by double in a Down’s fetus.

10. A 37-year-old lady, who is 16 weeks pregnant has noticed a small lump in her right
breast. Her mother and sister have both been treated for breast cancer in the past.
What is the best initial investigation to obtain a diagnosis?
A. Breast ultrasound
B. Mammography with fetal shielding
C. Fine needle aspiration cytology
D. Gadolinium enhanced magnetic resonance imaging (MRI)
E. Tumour markers like CEA, CA125

Answer: A
Breast ultrasound would be the first investigatin of choice and if cancer is strongly suspected, a
mammography would inform the clinician about the extent and spread of the lesion. Tumour
markers are not very reliable in pregnancy and MRI should not be used as a first line
investigation modality in pregnancy.

11. A 34-year-old pregnant lady, G2P1 has been diagnosed with ductal carcinoma of the right
breast (Stage 1). She is currently 22 weeks pregnant.
What is the initial treatment of choice for her?
A. Termination of pregnancy
B. Local mastectomy with reconstruction
C. Local mastectomy without reconstruction
D. Local radiotherapy
E. Single dose chemotherapy with trastuzumab

Answer: C
Termination of pregnancy is not a treatment at 22d weeks. Reconstructive surgeries should not
be performed in pregnancy. Radiotherapy is not the first line treatment and is more to prevent
relapses while trastumuzab is contraindicated in pregnancy. The initial treatment should be local
mastectomy only.

12. A 32-year-old Asian woman with a BMI of 35 at 14 weeks of gestation attends the
antenatal visit. You recommend vitamin D in pregnancy as per proctocol.
What is the daily recommended dose of vitamin D for this lady?
A. 10 iu
B. 10 mcg
C. 1000 iu
D. 1000 mcg
E. 20,000 iu
Answer: C
This is the recommended dose for women at high risk of vitamin D deficiency; her risk factors
are ethnicity and raised BMI. 10 mcg/day is the dose for all pregnant women. A dose of 20,000
iu cholecalciferol is used for treatment of known vitamin D deficiency.

13. High dose of vitamin D is recommended for pregnant women with ceratin risk factors.
What is the most important risk factor for vitamin D deficiency in a Caucasian woman
with low BMI?
A. Light skin pigmentation
B. Increased exposure to sunlight
C. Hypothyroidism
D. BMI <19
E. Immobility

Answer: E
Women with pigmented or covered skin, obesity and immobility are at a higher risk of vitamin D
deficiency.

14. A 35-year-old G4P3 is currently 16 weeks pregnant and has uncomplicated chronic
hypertension.
What should be the ideal target BP with treatment?
A. Less than 200/110
B. Less than 160/100
C. Less than 150/100
D. Less than 140/100
E. Less than 130/90

Answer: C

15. A 36-year-old G2P1 presents to the antenatal clinc. She had an emergency caesarean
section for sudden onset hypertension and placental abruption at 30 weeks in her previous
pregnancy. She is currently 20 weeks of gestation and enquires about further plan of fetal
monitoring in this pregnancy.
What is the most appropriate advice?
A. No extra monitoring is required
B. Uterine artery Doppler at 22 weeks
C. Seria scans starting from 24 weeks
D. Serial cardiotocograph monitoring from 28 weeks
E. Serial scans from 28 weeks

Answer: E
Serial scan from two weeks before last abruption gestation or serial scans from 28-30 weeks is
recommended in this case. The NICE recommendation is to carry out ultrasound fetal growth,
amniotic fluid volume assessment and umbilical artery Doppler velocimetry starting at between
28 and 30 weeks (or at least 2 weeks before previous gestational age of onset of pre-eclampsia if
earlier than 28 weeks) and repeating at 4 weekly intervals in women with previous:
- Severe pre-eclampsia
- Pre-eclampsia that needed birth before 34 weeks
- Pre-eclampsia with a baby whose birth weight was less than the 10th centile
- Intrauterine death

16. A 41-year-old primigravida with DCDA twins following a successful IVF treatment
presents with a booking BMI of 42 at 13 weeks of pregnancy and is seen in the antenatal
clinic.
What is the most appropriate medication that you would recommend?
A. Folic acid 5 mg/day
B. Vitamin D 10 mcg/day
C. Aspirin 75 mg/day
D. Metformin 500 mg TDS
E. Omega 3 fatty acid

Answer: C
Folic acid is recommended pre-conceptually and until 12 weeks of pregnancy. The correct dose
of Vitamin D dose in view of high BMI is 1000 mcg. BP check is recommended 3-4 weekly in
the second trimester at risk pregnancies. Weight loss is generally not recommended once a
woman is pregnant, healthy diet is recommended. Advise women with more than one moderate
risk factor for pre-eclampsia to take 75 mg of aspirin daily from 12 weeks until the birth of the
baby. Factors indicating moderate risk are:
- First pregnancy
- Age 40 yeears or older
- Pregnancy interval of more than 10 years
- Body mass index (BMI) of 35 kg/m2 or more at first visit
- Family history pre-eclampsia
- Multiple pregnancy

17. A 19-year-old primigravida at 34 weeks of pregnancy was admitted with a BP of 160/100


and no proteinuria. Her BP is currently controlled with labetalol 200 mg TDS. The fetal
growth and liquor volume is normal but she is very anxious about delivery.
What is the most appropriate advice regarding delivery?
A. Delivery with steroid cover after 34 weeks
B. Deliver at around 36 weeks
C. Caesarean section at 37 weeks
D. Deliver after 37 weeks
E. MgSO4, corticosteroids and delivery soon

Answer: D
Do not offer birth to women with chronic hypertension whose blood pressure is lower than
160/100 mmHg, with or without antihypertensive treatment, before 37 weeks. For women with
chronic hypertensiono whose blood pressure is lower than 160/110 mmHg after 37 weeks, with
or without antihypertensive treatment, timing of birth and maternal and fetal indications for birth
should be agreed between the woman and the senior obstetrician.
18. A primigravida at 36 weeks gestation is measuring large for dates. Ultrasound shows AC
>97th centile. GTT performed shows poorly controlled gestational diabetes.
What is the immediate management plan?
A. Give steroids
B. Start induction
C. Start hypoglycaemics
D. Wait and watch
E. Start sliding scale and deliver

Answer: C
Hypoglycaemic therapy should be considered for women with gestational diabetes if ultrasound
investigation suggests incipient fetal macrosomia (abdominal circumference above the 70th
percentile) at diagnosis.

19. A primigravida at 35 weeks of gestation presents with pain in the right hypochrondrium
and right side of her back. There is no history of nausea or vomiting, hypertension,
urinary symptoms and bowel problems. Vital signs: pulse-106, temperature-38.1, BP-
128/75. Abdominal examination is unremarkable. Chest is clear. Fetal monitoring is
normal. Urine shows 2+ leucocytes and 1+ blood. White cell count 16 x 109/L.
What is the most likely diagnosis?
A. Appendicitis
B. Cholecystitis
C. Pyelonephritis
D. Abruption
E. Right basal pneumonitis

Answer: C
Although the patient has no urinary symptoms, pyelonephritis is the most common infection in a
pregnant woman. Based on the raised WCC, fever and urinary leucocytes on microscopy, the
most likely diagnosis is pyelonephritis.

20. 24-year-old West African primigravida is 26 weeks pregnant. She calls the delivery suite
with symptoms of feeling unwell with diarrhea and vomiting for 3 days, which she
attributes to the food served on her flight from Nigeria 1 week ago.
What is the advice that you would give her?
A. Go to GP’s surgery
B. Present herself to A&E
C. Call for an ambulance
D. Call the infectious disese hot line
E. Come to delivery suite

Answer: C
Unwell patient from West Africa can be a possible case of Ebola. Transfer by ambulance after
alerting ambulance crew and the receiving hospital for appropriate precautions is the best option.
They should minimize contact with other people, therefore walking in to A&E or GP surgery or
delivery suite themselves isn’t appropriate. Contact with the infectious disease specialist is made
by GP or the first contact doctor and not by patient. The main points in the care of women with
suspected Ebola is
- Isoloation in a separate room
- Minimal staff contact
- Appropriate protective gear
- Talk but don’t touch
- Resuscitation of patient with caution
- No CTG or fetal monitoring
- No operative delivery

21. A patient who wishes to have cell free fetal DNA (cff DNA) testing enquires about how
soon it can be done?
What would you advise?
A. 1st trimester
B. Early 2nd trimester
C. Late 2nd trimester
D. 3rd trimester
E. Just before conception

Answer: A
The amount of cffDNA in maternal blood increases with gestational age. If the samples are taken
too early in pregnancy, false-negative results with Y chromosome or RhD testing become more
likely. For aneuploidy testing, at the current depth of sequencing, many protocols allow detection
only when the fetal DNA percentage is at least 4-5%. The commercially available aneuploidy
tests based on MPS are generally stated to be for pregnant women from 10 weeks of gestation; a
dating scan to establish gestational age before the sample is drawn is required.

22. A 32-year-old G3P2 with a history of an intrauterine death at 31 weeks in her last
pregnancy due to haemolytic disease of newborn is very anxious. She wants to know
about the method of testing her baby’s rhesus status with the least likelihood of
miscarriage.
What is her best option?
A. Fetal blood sampling
B. Cff DNA testing
C. Chorionic villus sampling
D. Cordocentesis
E. Amniocentesis

Answer: C? B
Cell free fetal DNA is a relatively new method, tested by venous maternal sample, non-invasive
to the fetus and therefore procedure related risk of miscarriage is absent.
23. A patient wishes to consider pregnancy after treatment for her breast cancer?
What is the most important predictor of a good prognosis?
A. Young age
B. Herceptin positivity
C. Estrogen receptor positivity
D. BRCA gene positive
E. Family history of treatable breast cancer

Answer: C
Estrogen receptor positive tumours have a better prognosis as compared to negative tumours.
Age and pregnancy itself are independent risk factors. However there can be delay in diagnosis
in pregnancy leading to advanced disease and hence worse prognosis. Similarly younger women
tend to have more estrogen receptor negative tumours that have worse prognosis.

24. A 33-year-old primigravida at 16 weeks gestation presents with a painless 2 cm breast


lump in left upper quadrant of breast. Ultrasound is suggestive of a malignant lesion.
What is the recommended procedure for diagnosis?
A. Fine needle aspiration cytology
B. MRI guided biopsy
C. Ultrasound guided biopsy
D. Lumpectomy
E. Gadolinium enhanced biopsy

Answer: C
Ultrasound guided biopsy is preferred over needle aspiration cytology in pregnant patients for
tissue diagnosis. Lumpectomy is not recommended for diagnosis. MRI guided biopsy and
Gadolinium enhanced biopsy are options that do not exist.

25. A 26-year-old primigravida who is a school teacher was exposed to chickenpox 7 days
ago. She is currently 19 weeks pregnant. She does not remember getting chickenpox rash
in childhood. Serum testing was organized by her GP and she is non-immune to varicella.
What is the single best advice for her?
A. VZIG will not be effective this late after exposure.
B. VZIG should be administered
C. Treat as infectious for up to 28 days
D. Check for features of FVS on 20 weeks scan
E. Oral acyclovir

Answer: B
VZIG can be given up to 10 days. Treat as infectious only if IG given. FVS feature may appear
after 5 weeks and not in 1 week. Acyclovir has no role in the management of exposure alone.

26. A 26-year-old primigravida who is a school teacher is non-immune to chickenpox. She


was exposed to chickenpox and had VZIG given at 19 weeks gestation. She had a further
exposure 2 weeks after the immunoglobulin was administered. The GP would like advice
above further management.
What is the best advice in this situation?
A. There is no need for further VZIG
B. Second dose of VZIG should administered
C. Repeat serology and then treat
D. Should work away from children
E. Oral acyclovir

Answer: A
A further exposure within 3 weeks does not warrant further VZIG.

27. A 23-year-old primigravida is booked with a midwife at 21 weeks. She reveals that she
had female circumcision done when she was a child. On examination the labia minora is
completely fused in the middle. You are to document the type of female genital
mutilation.
What type of female genital mutilation is this best described as?
A. Type 1
B. Type 2
C. Type 3
D. Type 4
E. Atypical presentation

Answer: C
When both labia minora are apposed in middle to create a seal it is type 1.

28. A mother attends your gynaecology clinic with her 5-year-old daughter and expresses her
wishes to have her circumscribed in accordance to her culture:
As per the recent guidelines on FGM what is your advice?
A. Respect their cultural belief and organize
B. Advise her to have it organized in her home country, as it is illegal in the UK
C. Involve child protection experts in care
D. Assist in the procedure as it lessens medical impact
E. Refer to the paediatric team

Answer: C

29. A 35-year-old P2 is 36 weeks pregnant. Clinically there is a suspicision of left calf DVT.
CTG is normal.
What is the next step in the immediate management?
F. Plan delivery
G. Therapeutic dose of tinzaparin
H. Prophylactic dose of tinzaparin
I. FBC, coagulation screen, LFTs and U&Es
J. Thrombophilia screen

Answer: C
Baseline bloods should be done before starting heparin low molecular weight therapy.
30. A 35-year-old P2 is 33 weeks pregnant. She arrives by ambulance in A&E in shock with
suspicision of a massive pulmonary embolism. Resuscitation has been initiated with good
response. Fetal heart is normal.
What is the next step in management?
F. Intravenous unfractionated heparin
G. Intravenous LMWH
H. Proceed to thoracotomy
I. Inferior vena cava filter
J. Surgical embolectomy

Answer: B? A
Collapsed, shocked patients need to be assessed by a team of experienced clinicians, including
the on-call consultant obstetrician, who should decide on an individual basis whether a woman
receives intravenous unfractionated heparin, thrombolytic therapy or thoracotomy and surgical
embolectomy. Intravenous unfractionated heparin is the preferred treatment in massive PE with
cardiovascular compromise.

31. A 33-year-old P1, who had a previous baby weighing 2.2 kg born at 39 weeks by LSCS
for fetal distress is in your antenatal clinic. She is currently 32 weeks pregnant. US shows
AC just above the 3rd centile. Umbilical artery Doppler is normal.
What is the appropriate management plan for her pregnancy?
A. Fundal height measurement by CMW fortnightly
B. Repeat growth and liquor volume scans three weekly
C. Repeat scans with uterine artery and DV Dopplers fortnightly
D. Repeat scans with umbilical artery and MCA Dopplers fortnightly
E. Twice weekly umbilical artery Dopplers

Answer: D
Fortnightly scans with umbilical artery and middle cerebral artery Dopplers is recommended.

32. What iis the best test to monitor glycemic control in a thalasaemia major patient with
established diabetes mellitus?
A. Fasting blood sugar
B. Postprandial blood sugar
C. HbA1c
D. Serum fructosamine
E. Glucose challenge test

Answer: D
Diabetes is common in women with thalassaemia due to pancreatic damage, iron overload,
genetic factors and autoimmunity.
Women with established diabetes mellitus should ideally have serum fructosamine
concentrations < 300 nmol/l for at least 3 months prior to conception. This is equivalent to an
HbA1c of 43 mmol/mol. Similar to women with diabetes without thalassaemia, an HbA1c of less
than 43 nmol/mol is associated with a reduced risk of congenital abnormalities. HbA1c is not a
reliable marker of glycaemic control as this is diluted by transfused blood and results in
underestimation, so serum fructosamine is preferred or monitoring.

33. A patient attends the preconception clinic. She has beta-thalassaemia and has regular
blood transfusions.
What is the most important systemic condition to rule out before she embarks on her
pregnancy?
A. Hypothyroidism
B. Diabetes
C. Cardiomyopathy
D. Cholecystitis
E. Gastric ulcers

Answer: C
Women are likely to get cardiomyopathy and cardiac arrhythmias with iron overload. This can
have a very negative impact on her pregnancy depending upon the severity of cardiac function.
All women should be assessed by a cardiologist with expertise in thalassaemia and/or iron
overload prior to embarking on a pregnancy. An echocardiogram and an electrocardiogram
(ECG) should be performed as well as T2* cardiac MRI. The aim is for no cardiac iron, but this
can take years to achieve so care should be individualized to the woman. Therefore the aim is for
cardiac T2* >20 ms wherever possible as this reflects minimal iron in the heart. A T2* <10 ms is
associated with an increased risk of cardiac failure. A reduced ejection fraction is a relative
contraindication to pregnancy and the management should be the subject of multidisciplinary
discussions involving a cardiologist with experience of cardiac pathology in pregnancy, a
maternal medicine specialist, a haematologist and an obstetric anaesthetist.
Liver and gall bladder (and spleen if present) ultrasound should be used to detect cholelithiasis
and evidence of liver cirrhosis due to iron overload or transfusion-related viral hepatitis.
Cholelithiasis is common in women with thalassaemia due to the underlying haemolytic anaemia
and they may develop cholecystitis in pregnancy. Liver cirrhosis and active hepatitis C (HCV)
may run a more complex clinical course during pregnancy. Women who are HCV RNA-positive
should be reviewed by their hepatologist preconceptually. Women who have any evidence of
cirrhosis, either due to previous hepatitis or as a consequence of severe hepatic iron loading
should be reviewed by a hepatologist.

34. A 38-year-old lady with primary infertility and major thalassemia is seeking fertility
treatment. You recommend ovulation induction after investigations. In order to optimize
the preconception status you perform some blood tests.
What is the most important test that determines a safe pregnancy in this situation?
A. Target liver iron of <7 mg/g
B. A platelet count of more than 150 x 109/L
C. Normal vitamin D levels
D. Normal bone mineral density
E. Normal bilirubin levels

Answer: A
A target liver iron of less than 7 mg/g dry weight (dw) is recommended because iron chelation is
discontinued during pregnancy and therefore tansfusional iron burden and the risk of iron
overload-related complications increases. If liver iron exceeds the target range, a period of
intensive preconception chelation is required to optimize liver iron burden. If liver iron exceeds
15 mg/g (dw) prior to conception, the risk of myocardial iron loading increases so iron chelation
with low-dose desferrioxamine should be commenced between 20 and 28 weeks under guidance
from the haemoglobinopathy team. Cholelithiasis is common in women with thalassemia due to
the underlying haemolytic anaemia and they may develop cholecystitis in pregnancy. Liver
cirrhosis and active hepatitis C (HCV) may run a more complex clinical course during
pregnancy. Women who are HCV RNA-positive should be reviewed by their hepatologist
preconceptually. Women who have any evidence of cirrhosis, either due to previous hepatitis or
as a consequence of severe hepatic iron loading, should be reviewed by a hepatologist.

35. What is the approximate incidence of breech presentation at 28 weeks of gestation?


A. 4%
B. 8%
C. 10%
D. 15%
E. 20%

Answer: E
It is 20% at 28 weeks and reduces to 3-4% at term.

36. A 20-year-old primigravida presents to the delivery suite with rupture of membranes and
labour pains for one hour. She is currently 32+4 weeks pregnant and a scan 5 days ago
showed flexed breech with mile oligohydramnios. The estimated baby weight according
to the last scan was 2050 grams. She has regular contractions and the cervix is 5
centimeters dilated, fully effaced with breech presentation at 0 station. She is transferred
to theatre. On examination the breech has delivered with fetal head inside with a tight
cervix.
What is the best intervention for a safe delivery in this situation?
A. Injection of terbutaline
B. Emergency caesarean section and use lower segment incision
C. Emergency section and use vertical midline uterine incision
D. Lateral cervical incisions
E. Delivery head by Burns Marshall technique

Answer: D
In a preterm fetus when the after-coming head is entrapped, head is not going to be delivered out
of an inadequately dilated cervix. The best option at this time would be to give a small cervical
incision avoiding 3 and 9 o’clock positions and maintaining suprapubic pressure to maximize the
flexion of head deliver by Mauriceau-Smellie-Veit technique.

37. A 22-year-old second gravida complains of slightly increased vaginal discharge for the
past one week. She is currently 14 weeks pregnant and is anxious as she has had a
miscarriage in her last pregnancy at 16 weeks following ruptured membranes. This was
followed by induction of labour after 72 hours for anhydramnios.
On speculum examination you find the cervix is long and closed with no evidence liquor.
Transvaginal ultrasound measures the cervical length as 30 mm.
What is the best management for her?
A. Consider a MacDonald’s cervical cerclage
B. Reassure her and call her back for repeat scan
C. Take a high vaginal swab and reassure her
D. Consider a Shirodhkar cerclage
E. Admit her for observation

Answer: C
A history indicated cerclage should be applied only when there are more than two mid-trimester
losses or preterm deliveries. McDonald’s technique is a transvaginal cervical suture without
bladder reflection and Shirodhkar is a high vaginal stitch after bladder reflection. Ultrasound
indicated stitch should be applied only if the cervical length is less than 25 mm in a woman with
a history of two second trimester births or miscarriages and not on evidence like funneling. There
is an increased risk of second trimester miscarriages when there is bacterial vaginosis. It is
advisable to take swabs when there is an increased vaginal discharge.

38. What would be a definitive indication for a prophylactic cervical cerclage in pregnant
women?
A. Twins conceived after treatment of infertility
B. Triplets and higher pregnancy orders
C. A previouis history of cone biopsy
D. History of septum resection
E. History of three or more preterm deliveries

Answer: E
39. A second gravida, currently 14 weeks pregnant, wishes to seek information regarding
Varicella. Her 5-year-old son has developed vesicular rash over the past 24 hours and has
been diagnosed as suffering from Varicella by his paediatrician. She is concerned above
the effects that it may have no her present pregnancy. She has not noticed any rash on her
body at present but is feeling lethargic.
What would you advise the best management plan for her?
A. Get a VZV- IgG from booking serum
B. Get a VZV- IgM from a fresh sample
C. Receive a shot of VZ immunoglobulin
D. Start on oral acyclovir tablets to reduce complications
E. Receive varicella vaccine injection

Answer: A
If she is already immune to varicella as suggested by a positive IgG, she need not worry. If for
some reason booking serum is not available, a fresh sample may be used or doing IgG.
Immunoglobulin or acyclovir is not indicated unless the disease develops. Varicella vaccine is a
live- attenuated vaccine and is contraindicated in pregnancy. If she is non immune then she
should be offered VZIG. The best answer in this question is to test the immunity first.

40. A 24-year-old midwife has just developed a vesicular rash on her abdomen which is
intensely itchy. She remembers seeing a patient of Varicella in her surgery last week. She
is currently in her first pregnancy and is 37 weeks pregnant. Her antenatal period has
been uneventful so far.
What would you advise her?
A. Watchful expectancy
B. Oral acyclovir therapy
C. Intravenous acyclovir therapy
D. Induction of labour
E. Immediate caesarean delivery

Answer: C
After 36 weeks, the risk of complications to mother and the transmission rates to the fetus up
especially if labour starts within 10 days of development of rash. VZIG may also be administered
to the mother and the neonate if she delivers within 7 days.

41. A 28-year-old primigravida Mrs Smith is waiting to see you in the antenatal clinic. She is
20 weeks pregnant, with an uneventful antenatal period so far. Her combined NT risk was
low. On the anomaly scan, there are two hyperechogenic foci seen in left ventricle. Rest
of the anomaly scan is normal. Mrs Smith would like to discuss the scan findings with
you.
Based on the above information what would be your advice?
A. Refer her for fetal ECHO
B. Counsel her for amniocentesis
C. Reassure her
D. Rescan in 2-3 weeks
E. Request a TORCH test

Answer: C
In the absence of any other soft markers and a low risk for trisomy, nothing more needs to be
done at this stage.
Source of information: www.fetalanomaly.screening.nhs.uk.
NHS screening for fetal anomalies: Normal variant screening recommend the following:
Women who are found to be ‘low risk’ through testing in either first or second trimesters, or who
have declined screening for Down’s syndrome should not be referred for further assessment of
chromosomal abnormality even if normal variants such as the examples below (whether single or
multiple) are seen at the 18+0 to 20+6 weeks fetal anomaly screening scan
- Choroid plexus cyst(s)
- Dilated cisterna magna
- Echogenic foci in the heart
- Two vessel cord
The appearance listed below (previously classified as “markers”) are examples of findings which
should be reported and the woman referred for further assessment and treated as for any other
suspected fetal anomaly
Structure Referral criteria
1. Nuchal fold Greater than 6 mm
2. Ventriculomegaly Atrium greater than 10 mm
3. Echogenic bowel With density equivalent to bone
4. Renal pelvic dilatation AP measurement greater than 7 mm
5. Small measuremetns compared to dating Significantly less than 5th centile on national
scan charts
Women who have not had Down’s screening (booked too late or are in a part of the UK in which
this is not available) should have counseling based on maternal age and/or family history not on
whether normal variants are found during scanning.

42. A 20-year-old has an anomaly scan at 21+6 weeks of gestation. This shows that the fetus
has severe congenital anomalies. Termiantion of pregnancy is commenced and the fetus
is delivered live.
How should the birth be registered/reported as?
A. Live birth
B. Miscarriage
C. Stillbirth
D. Does not need registration
E. Reported as termination

Answer: A

43. A 32-year-old P1 with no risk factors and an uneventful last pregnancy is seen by you at
17 weeks with the results of quadruple test. The results are:
Estriol is 0.5 MoM
hCG is 1.5 MoM
AFP is 1.0 MoM
Inhibin A is 1.4 MoM
Risk for Trisomy 21, 18, 13 and NTDs is low
How would you counsel this lady?
A. Reassure that risk of Trisomy 21, 18, 13 and NTD is low
B. Increased risk of pre-eclampsia
C. Increased risk of SFD
D. Increased risk of stillbirth
E. Amniocentesis is indicated

Answer: C
NT (nuchal translucency)
This is the most important ultrasound marker and the most influential on risk calculation. No
pathologies associated with a NT below 1 MoM are known. The thicker the NT is, the worse
fetal prognosis is. Pathological values of NT usually range between 1.8-2 MoM or measure
greater than 3 mm (regardless of the MoM)
PaPP-A (pregnancy-associated plasma protein A)
Overall, fetuses with chromosomal abnormalities have low PAPP-A values. Values lower than
0.4 MoM increase the risk of chromosomal anomaly.
Free beta hCG or hCG (total human chorionic gonadotrophin or its free beta fraction)
Fetus with chromosomal abnormalities can present altered free beta hCG or hCG values. Overall,
in Trisomy 21 these values are higher, those higher than 2.5 MoM indicating a possible
pathology. In Trisomies 13 and 18, these values are generally low, with suspicious values being
those below 0.4 MoM.
AFP (alpha-fetoprotein)
High levels of AFP, higher than 2.5 MoM, may indicate fetal malformation (spina b ifida,
anencephaly, etc.).
Low levels of AFP, lower than 0.4 MoM, are observed in certain chromosomopathies. Hence, in
trisomies 21 and 18 AFP may present low values whereas in trisomy 13 it can be slightly higher
than normal.
uE3 (unconjugated estriol) Values lower than 0.5 MoM can indicate possible trisomy 21, 13 or
18.
Inhibin A
When its values are greater than 2.5 MoM, they may be indicative of trisomy 21 or 13.

44. A HIV positive pregnant woman is keen on a vaginal birth.


What factor will you consider before advising her a vaginal birth to reduce the risks of
transmission to the baby?
A. Plasma viral load <200 copies/ml
B. CD4 count <200 cells/cumm of blood
C. Plasma viral load <50 copies/ml
D. On zidovudine therapy
E. On HAART

Answer: C
A vaginal birth has a small risk of viral transmission to the neonate if the maternal viral load is
<50 copies/ml. Hence, this cut off value is the most important factor.

45. When should HIV positive pregnant women with no predisposing factors for diabetes be
screened for gestational diabetes?
A. If they have recurrent infections
B. If they are on zidovudine
C. If they are on HAART
D. If vaginal birth is planned
E. If they have hepatitis co-infection

Answer: C
46. HIV positive women on HAART are predisposed to certain conditions in late pregnancy.
What increased risk would you warn these women about?
A. PPROM
B. Preterm labour
C. Caesarean section
D. Asymptomatic vaginal infection
E. Hypertension in pregnancy

Answer: B

47. A HIV positive pregnant woman is having a planned vaginal birth. At full dilatation,
vertex is direct OA at +2 station and the midwife is concerned about a pathological CTG.
What is the most appropriate management in this situation?
A. Fetal blood sampling
B. Fetal scalp electrode monitoring
C. Ventouse delivery
D. Outlet forceps delivery
E. Caesarean section

Answer: D
Outlet forceps delivery is most appropriate.

48. A 33-year-old primigravida presents at 33+4 weeks of gestation with persistently raised
BP between 140/90 to 145/99 for one week. There is no proteinuria and SFH is normal.
What will the most important further management include?
A. Checking BP weekly
B. Checking BP twice weekly
C. Admit and do a BP profile
D. Do BP monitoring weekly with blood tests fortnightly
E. Offer delivery at 34 weeks

Answer: A
Mild gestational hypertension requires monitoring no more than once a week if developed after
32 weeks of gestation.

49. A 33-year-old primigravida presents at 27 weeks of gestation with persistently raised BP


between 140/90 to 145/99 for one week. There is no proteinuria and SFH is normal.
What is the most important further management?
A. Checking BP weekly
B. Checking BP twice weekly
C. Admit and do a BP profile
D. Do BP monitoring weekly with blood tests fortnightly
E. Offer delivery at 34 weeks

Answer: B
For mild hypertension before 32 weeks of gestation, measure BP twice weekly.
Best option would actually be test for proteinuria and measure BP twice weekly (which is not an
option in this question) and therefore second best option is used.
50. A 33-year-old primigravida presents at 34 weeks of gestation with raised BP >160/110.
There is no proteinuria and SFH is normal. She is admitted to the hospital and started on
antihypertensive therapy.
What is the most appropriate further management plan?
A. Keep admitted, control BP and delivery at 36 weeks of gestation
B. Admit, BP control and monitoring at least 4 times/day
C. Admit, BP control and monitoring 4 times/day, daily proteinuria, weekly blood tests
till delivery
D. Admit, BP control and monitoring 4 times/day, daily proteinuria twice weekly blood
tests
E. Give MgSO4 and deliver

Answer: D? C
Measure blood pressure at least 4 times a day. Test for proteinuria daily using an automated
reagent-strip reading device or urinary protein:creatinine ratio. Test renal function, electrolytes,
FBC, transaminases and bilirubin at presentation and then monitor weekly.

51. What is the single best practice for a successful VBAC?


A. Spontaneous labour
B. Previous vaginal birth
C. Previous caesarean section at full dilatation
D. BMI <30
E. Epidural analgesia

Answer: B
Previous vaginal birth is the most predictive factor for success of vaginal birth after caesarean
section. Epidural analgesia increase the chances of successful vaginal birth.

52. A 33-year-old P1 presents in clinic with a history of previous caesarean section for
breech presentation. Her booking BMI is 34 and she is currently at 40+10 weeks
gestation. She is very keen for vaginal birth.
What are her chances of a successful vaginal birth?
A. 10%
B. 25%
C. 55%
D. 75%
E. 90%

Answer: D
Women considering their options for birth after a single previous caesarean section should be
informed that overall chances of successful planned VBAC are 72-76%.

53. In a second trimester screening ultrasound, which soft markers has the greatest likelihood
of predicting a Trisomy 21?
A. Echogenic cardiac focus
B. Short femur
C. Increased nuchal translucency
D. Absent or hypoplastic nasal bone
E. Echogenic bowel

Answer: D
Nasal bone is absent in majority (75%) of Down’s syndrome at 12 weeks scan however if absent
in the second trimester (15-2 weeks scan) can be a marker for Down’s syndrome. Likelihood
Ratios for isolated markers for trisomy 21 are 7.5 for absent humerus, 2.7 for short femur, 17 for
increased Nuchal Translucency, 6.1 for echogenic bowel and 2.8 for echogenic cardiac focus.

54. There is a risk of major fetal malformations with most antiepileptic.


Which antiepileptic has the least risk of malformation in the fetus?
A. Sodium valproate
B. Phenytoin
C. Carbamazepine
D. Levetiracetam
E. Lamotrigine

Answer: E
The results are similar to the findings of a meta-analysis published in 2008 including registry and
cohort data up until May 2007. The incidence (95% confidence intervals) of malformations for
monotherapy was as follows:
Carbamazepine 4.6% (3.5-5.8%)
Lamotrigine 2.9% (2-3.8%)
Phenobarbitol 4.9% (3.2-6.6%)
Phenytoin 7.4% (3.6-11.1%)
VPA (Valproic acid) 10.7% (8.2-13.2%). As a single agent, VPA consistently has the highest
rate of malformations, and several investigations have reported a dose-response relationship
between valproate and major malformation.

55. A young pregnant woman was admitted via A&E with severe pain abdomen, fever and
vomiting. She gives a history of travel to India two months ago. Her blood film
examination shows that 5-6% erythrocytes are infected with signet rings of plasmodium
falciparum.
What is the drug of choice for the treatment?
A. Intravenous primaquine
B. Intravenous chloroquine
C. Intravenous quine
D. Intravenous artesunate
E. Intravenous clindamycin

Answer: D
Primaquine should not be used in pregnancy. Chloroquine and clindamycin are reserved for
uncomplicated Malaria and here since more than 2% erythrocytes are infected she qualifies for
severe plasmodium falciparum malaria infection. Intravenous Artesunate is the first choice for
her followed by Quinine if artesunate is not available. Oral medication should not be used in her
due to vomiting.

56. A pregnant woman is being treated for malaria with quinine.


What is the most common side effect of quinine therapy?
A. Hypoglycemia
B. Hypocalcaemia
C. Hypomagnesaemia
D. Hypokalemia
E. Hyponatremia

Answer: A
Hypoglycemia is common and may cause fetal distress.

57. What is the cause of the commonest form of malaria in the UK?
A. Plasmodium vivax
B. Plasmodium falciparum
C. Plasmodium ovale
D. Plasmodium malariae
E. Plasmodium Knowlesi

Answer: B
P. falciparum malaria is commonest and most dangerous of all types in terms of complications.
In UK it is mostly due to imported infections from Africa subcontinent. P. vivax is commonly
imported from Indian subcontinent.

58. A pregnant woman, at 34 weeks of gestation diagnosed with iron deficiency anemia
refractory to oral therapy was asked to come for an intravenous iron drip to day
assessment unit. Ten minutes after starting the iron infusion, she starts complaining of a
choking sensation and difficulty in breathing. You are the registrar on duty and on
reaching the patient you quickly stop the infusion and call for cardiac arrest. In the
meantime, she starts gasping with shallow breaths.
What is the drug of choice in this situation?
A. 500 µg of 1:1000 adrenaline administered intravenously
B. 500 µg of 1:1000 adrenaline administered intramuscularly
C. 500 µg of 1:10,000 adrenaline administered intramuscularly
D. 500 µg of 1:10,000 adrenaline administered intravenously
E. 500 µg of 1:10,000 adrenaline administered intracardiac

Answer: B
Adrenaline (0.5ml) pushed intramuscularly is the drug of choice for anaphylaxis followed by
injection chlopheniramine 10 mg and hydrocortisone 200 mg given as slow IV or
intramuscularly.

59. A 30-year-old gravida 3 with a BMI 27 at 32/40 has come to you for a diagnosis about
her mode of delivery. She has a 5-year-old son born vaginally and 2 years back she
delivered another baby boy by caesarean section for breech presentation. Her current
pregnancy is uneventful with cephalic presentation and she is very keen for a vaginal
delivery.
What are the chances of a successful vaginal delivery?
A. 65-70%
B. 70-75%
C. 75-80%
D. 80-85%
E. 85-90%

Answer: E
The success rate is higher due to a previous normal delivery and a CS for non-recurrent
condition.

60. What is single best predictor of vaginal birth after a caesarean section?
A. History of previous uncomplicated caesarean section
B. History of average sized babies
C. History of a previous vaginal birth
D. History of previous uneventful postoperative period
E. History of carrying a female baby in current pregnancy

Answer: C

61. A 39-year-old diabetic woman at 37/40 weeks attends the labour ward with complaints of
leaking clear fluid per vaginuum. She is booked for an elective caesarean section in one
week’s time for a previous third degree tear. On speculum examination there is clear
liquor seen, cervix is long and closed and the admission CTG is normal.
What is the next course of action?
A. Reassure and do a caesarean section immediately
B. Give single dose of steroids and do a CS after 12 hours
C. Reassure and send home
D. Give full dose steroids and perform a caesarean
E. Give IV antibiotics and perform a caesarean

Answer: D
Since there is no emergency for her caesarean section, it is best to give a full course of steroid
despite her diabetic status. For those diabetic patients on insulin, sliding scale should be started
while administering steroids. All patients who have a caesarean section done before 38+6 days of
gestation should receive antenatal corticosteroids.

62. Mrs. Smith who is 35/40 weeks pregnant has been complaining of loss of fetal
movements for the past two days. A Doppler scan shows reversed end diastolic flow in
the umbilical vessels. The estimated fetal weight by ultrasound is 1.6 kg. She is not in
labour and the cervix is unfavourable.
What is the next course of action?
A. Perform a caesarian section immediately
B. Reassure and proceed for induction of labour
C. Give a single dose of steroid and do a caesarean section
D. If the CTG is normal give two doses of steroids and then do a CS
E. Give two doses of steroids, MgSO4 and deliver

Answer: D
For growth restricted babies, two doses (single course) of steroids is recommended for lung
maturity and MgSO4 for neural protection is recommended if the baby is < 30 weeks.

63. An Asian primigravida, 32/40 presents for routine antenatal check-up. She complains of
tiredness and looks slightly pale. Her haemoglobin is 10.5 g% despite an Hb of 13 g% at
the time of booking. All her other booking investigations were normal. She admits that
she is a vegetarian.
What is the best course of action to improve her Hb is:
A. Balanced diet
B. Oral iron
C. Intravenous iron sucrose
D. Intravenous iron dextrose
E. Recombinant human erythropoietin injection

Answer: B
A haemoglobin less than or equal to 10.5 requirs oral iron. Parenteral iron is not indicated with
an Hb of 10.5 g% would be used only when compliance or absorption is in doubt.

64. A 24-year-old pregnant woman has been admitted to HDU with a diagnosis of Dengue
haemorrhagic fever. She is currently 29 weeks pregnant and her blood parameters show a
haemoglobin of 11 g%, platelet count of 60 x 109/l, fibrinogen levels at 1.3g/l. A
decision is made to deliver her baby by caesarean section.
What is the blood product that she may require during caesarean section?
A. Whole blood
B. Platelet concentrate
C. Fresh frozen plasma
D. Cryoprecipitate
E. Packed red cells

Answer: B
In pregnant patients the margin of safety for platelet concentration should be kept at 75 x 109/l,
the rest of the parameters given are all within the normal range at present.

65. A 32-year-old second gravida presents to the labour ward with complaints of leaking for
the last 6 hours. She is currently 24 weeks pregnant and on speculum examination you
find a clear pool of fluid in the posterior fornix. You explain the prognosis to the mother
and recommend admission.
What is the most appropriate test to monitor her condition after admission?
A. Maternal full blood count every alternate day
B. Maternal CRP repeated twice weekly
C. Maternal high vaginal swabs repeated every alternate day
D. Maternal pulse and temperature repeated three to four times per day
E. Maternal vaginal examination done every day to check for cervical dilatation.

Answer: D
Maternal full blood count and CRP are not very accurate indicators for chorioamnionitis or fetal
infection. High vaginal swabs should be done once on admission and per vaginal examination
should be only repeated when there is a strong suspicion of labour. Maternal tachycardia and
pyrexia may be the first indicators of an impending chrioamnionitis and termination of
pregnancy 4-6 hourly observation is recommended.

66. A 35-year-old primigravida has been admitted at 27 weeks gestation with confirmed
PPROM. Her blood counts on admission have been normal and high vaginal swab is
culture negative.
What is the most appropriate treatment for her?
A. No antibiotics till cultures suggest a growth
B. Commence oral erythromycin
C. Commence oral augmentin
D. Commence low dose aspirin
E. Commence LMWH

Answer: B
The current guidelines recommend erythromycin 250 mg qds for at least 10 days for prevention
of chorioamnionitis even when cultures are negative. Augmentin is especially contraindicated in
pregnancy as it may predispose the baby to necrotizing enterocolitis.
Twenty-two trials involving over 6000 women with PPROM before 37 weeks of gestation were
included in a meta-analysis. The use of antibiotics following PPROM is associated with a
statistically significant reduction in chorioamnionitis (RR 0.57; 95% CI 0.37-0.86). Neonatal
infection was significantly reduced in the babies whose mothers received antibiotics (RR 0.68;
95% CI 0.53-0.87). There was also a significant reduction in the number of babies with an
abnormal cerebral ultrasound scan prior to discharge from hospital (RR 0.82; 95% CI 0.68-0.98).
There was no significant reduction in perinatal mortality, although there was a trend for
reduction in the treatment group.

67. A 20-year-old primigravida, 24 weeks gestation has come for her routine antenatal visit
and seems concerned that she has never felt fetal movements so far. She is a migrant, a
heavy smoker and consumes alcohol regularly. She has had one ultrasound scan at 17
weeks which showed an anterior placenta, with a normal looking fetus with slightly
reduced liquor.
With the given history what is the condition the fetus could have?
A. Renal agenesis
B. Holoprosencephalic fetus
C. Dandy-Walker malformation
D. Congenital myasthenia
E. Polycystic kidneys
Answer: D
The possibility of neuromuscular problems should be kept in mind if a pregnant woman is unable
to perceive fetal movements till 24 weeks of gestation. Rest of the anomalies have no concern
with the reduced fetal movements and can easily be picked up on an anomaly scan.

68. An 18-year-old lady in her first pregnancy presents to the labour ward at 26 weeks with
fits. She is known to have epilepsy in childhood but not had any fits for 10 years. On
examination there are tonic seizures, no cyanosis, resistance to passive eye opening,
down going plantar reflexes and persistence of positive conjunctival reflex.
What is the most likely diagnosis?
A. Epileptic seiqures
B. Eclamptic seiqures
C. Gestational epilepsy
D. Pseudoepilepsy
E. Hypercalcemia

Answer: D
Pseudo fits or pseudo epilepsy are fits characterized by the prolonged repeated seizures without
cyanosis, resistance to passive eye opening, down going plantar reflexes and persistence of
positive conjunctival reflex. Most patients are known to have epilepsy.

69. Seizures in pregnancy can be caused by hyperemesis gravidarum.


What is most likely pathophysiology?
A. Hypoinsulinaemia
B. Hypothyroidism
C. Hypovitaminosis
D. Hypokalaemia
E. Hyponatremia

Answer: E
Causes of fits in hyperemesis gravidarum are electrolyte disturbances such as hypocalcaemia,
hyponatremia, hypoglycaemia.
Other causes of seizures in pregnancies:
- Eclampsia
- Cerebral vein thrombosis
- Thrombotic thrombocytopenic purpura
- Cerebral infarction
- Drug and alcohol withdrawal
- Hypoglycemia (diabetes, hypoadrenalism, hypopituitarism, liver failure)
- Hypocalcemia (magnesium sulphate therapy, hypoparathyroidism)
- Hyponatremia (hyperemesis)

70. A pregnant mother is on valproate, carbamazepine and phenytoin for uncontrolled


epilepsy?
What is the risk of fetal anticonvulsant syndrome?
A. 6%
B. 15%
C. 25%
D. 50%
E. 75%

Answer: D
The risk of teratogenicity with 1 anticonvulsant is 6-7%, using 2 is about 15% and using 3 drugs
is as high as 50%.

71. Vitamin K is usually administered to pregnant women on hepatic enzyme inducing drugs
from 35 weeks of pregnancy. This is because of the reduction in vitamin K dependent
clotting factors.
Which clotting factor remains unchanged as it is not vitamin K dependent?
A. Factor 12
B. Factor 10
C. Factor 2
D. Factor 7
E. Factor 9

Answer: A
The hepatic enzyme inducers reduce the vitamin K dependent clotting factors like 2,7,9,10 thus
increasing the risk of haemorrhage disease of the newborn.

72. A third gravida with breech presents in the triage ward with leaking per vaginum without
any labour pains at 32 weeks. On examination, FHR is 140 bpm, cervix is 4 cm dilated
with a loop of cord lying in the vagina with good cord pulsations. You decide to perform
a category 1 caesarean section.
What is the best intervention to prevent fetal asphyxia until she is transferred to theatre?
A. Start intranasal oxygen at 5-10 l/min
B. Start intranasal oxygen at 10-15 l/min
C. Put her in lithotomy position
D. Put her in left lateral position with a pillow under her buttocks
E. Manually replace the cord in the uterus

Answer: D
Fetal heart rate is good at this time and unnecessary cord handling may lead to vasospasm. Best
practice is to reduce pressure on the cord by either pushing the head or buttock up or pushing the
presenting part by tilting the patient sideways and lifted up buttocks. Filling up the urinary
bladder and tocolysis till surgery may also help.

73. In mothers who have active herpes type 2 lesions there is a risk of transmission to the
baby. Certain factors increase the risk of transmission.
Which factor has the highest risk of neonatal herpes?
A. Preterm rupture of membranes within 2 weeks of delivery
B. If a primary infection occurs in the third trimester
C. If there are transplacental neutralizing maternal antibodies
D. If scalp electrodes are used
E. If vaginal delivery occurs

Answer: B
Factors associated with transmission include the type of maternal infection (primary or
recurrent), the presence of transplacental maternal neutralizing antibodies, the duration of rupture
of membranes before delivery, the use of fetal scalp electrodes and the mode of delivery. The
risks are greatest when a woman acquires a new infection (primary genital herpes) in the third
trimester, particularly within 6 weeks of delivery, as viral shedding may persist and the baby is
likely to be born before the development of protective maternal antibodies.

74. A woman presents at 36 weeks of pregnancy and has acquired recurrent herpes infection.
The patient is very anxious about transmitting this to her baby. She would like to have
more information about reducing the risk.
What would your advice be?
A. The risk of neonatal herpes is about 20% with a vaginal delivery
B. Oral acyclovir reduces the risk of infection to the baby
C. Sequential PCR to detect viral shedding can help advise mode of delivery
D. Dose of oral acyclovir should be 400 mg three times instead of the twice daily dose
E. Caesarean section is advisable based on the risk of neonatal infection

Answer: D
Women with recurrent genital herpes should be informed that the risk of neonatal herpes is low,
even if lesions are present at the time of delivery (0-3% for vaginal delivery). Vaginal delivery
should be anticipated in the absence of other obstetric indications for caesarean section. Daily
suppressive acyclovir 400mg three times daily should be considered from 36 weeks of gestation.
This increase from the standard suppressive dose of 400 mg twice daily is recommended in view
of the greater volume of distribution of the drug during pregnancy.
There is insufficient evidence to determine whether this reduces the incidence of neonatal
herpes; however, it reduces viral shedding and recurrences at delivery so may reduce the need for
caesarean section.
Sequential PCR culture during late gestation to predict viral shedding at term, or at delivery to
identify women who are asymptomatically shedding HSV, is not indicated.
There is no increased risk of preterm labour, preterm pre-labour rupture of membranes or fetal
growth restriction associated with women seropositive for HSV. The incidence of congenital
abnormalities is not increased in the presence of recurrent genital herpes infection.

75. What is the major form of vitamin D in human body?


A. Cholecalciferol
B. Ergocalciferol
C. 25, hydroxyl vitamin D
D. 7, dehydrocholesterol
E. 1,25dihydroxyl vitamin D

Answer: C
Though 1,25 dihydroxyl vitamin D is the biologically more active form, 25 hydroxy vitamin D is
the major circulating form but has lower activity.

76. What is the most prevalent complication of low maternal serum calcium in pregnancy?
A. Pre-eclampsia
B. Tetany
C. Bradycardia
D. Low cardiac ejection fraction
E. Intrauterine growth restriction

Answer: A
Pre-eclamspia and neonatal hypocalcaemia are two most prevalent complications of maternal
hypocalcemia. Low fetal birth weight and growth restriction is more related to vitamin D
deficiency.

77. A woman who is HIV antibody positive with low viral load aims for a vaginal birth. She
has a history of recurrent genital herpes.
Based on this what should you advise?
A. Elective caesarean section
B. Oral acyclovir should be started at 32 weeks of gestation
C. Oral acyclovir 2 weeks before the caesarean delivery
D. Lower risk of transmission of HIV to the baby
E. Higher risk of fetal growth restriction with acyclovir

Answer: B
There is some evidence that HIV antibody positive women with genital HSV ulceration in
pregnancy are more likely to transmit HIV infection independent of other factors.
Women who are HIV antibody positive and have a history of genital herpes should be offered
daily suppressive acyclovir 400 mg three times daily from 32 weeks of gestation to reduce the
risk of transmission of HIV infection, especially in women where a vaginal delivery is planned.
Starting therapy at this earlier gestation than usual should be considered in view of the increased
possibility of preterm labour in HIV positive women.
The mode of delivery should be in line with BHIVA HIV in pregnancy guideline
recommendations according to obstetric factors and HIV parameters such as HIV viral load.
There is currently no evidence to recommend daily suppressive treatment of HSV for HIV
antibody positive women who are HSV-1 or -2 seropositive but have no history of genital herpes.

78. A primigravida at 37+1 weeks of gestation with an uneventful pregnancy presents with
prelabour rupture of membranes. She had a vaginal swab done at 34 weeks that was GBS
positive. The following advice is appropriate:
A. Offer caesarean section at 39 weeks
B. Offer antibiotic prophylaxis when in labour
C. Offer immediate induction of labour and IV antibiotics
D. Take swabs again and wait for 24 hours for spontaneous labour
E. Offer immediate caesarean section
Answer: C
Evidence level 3: Immediate induction of labour and IAP should be offered to all women with
prelabour rupture of membranes at 37+0 weeks of gestation or more.
The NICE guideline on induction of labour recommends that all women with prelabour rupture
of membranes at term (37 weeks + 0 days of gestation or greater) should be offered immediate
induction of labour, or induction after 24 hours. If GBS colonization was identified earlier in the
pregnancy (by a swab taken for other reasons), immediate induction of labour and IAP should be
offerd.

79. A lady is diagnosed with a confirmed intrauterine fetal death (IUFD). She is known to be
GBS positive.
Which drug is not indicated in the intrapartum care in this woman?
A. Mifepristone
B. Prostaglandin analogues for induction of labour
C. Broad spectrum antibiotics if there is sepsis
D. Diamorphine for pain relief
E. Antibiotic prophylaxis for GBS, as mother known to be positive

Answer: E
- Women with sepsis should be treated with intravenous broad-spectrum antibiotic therapy
(including antichlamydial agents)
- Routine antibiotic prophylaxis should not be used.
- Intrapartum antibiotic prophylaxis for women colonized with group B Streptococcus is
not indicated
- Diamorphine is preferred over pethidine for pain releif

80. Dopamine agonist for lactation suppression is indicated in a woman diagnosed with a
stillbirth.
Which medical disorder would be a contraindication for dopamine agonist?
A. Thyroid disease
B. Systemic lupus erythematosus
C. Pre-eclampsia
D. Diabetes
E. Myasthenia gravis

Answer: C
Cabergoline is preferred over bromocriptine. It should be avoided in women with hypertension
or preeclampsia.

81. A 28-year-old primigravida with known SLE at 35 weeks presents with acute abdominal
pain. She is Rhesus B negative. An acute abruption is suspected.
Which blood test is appropriate to check if Anti-D is indicated?
A. Free fetal DNA for fetal blood group
B. Kleihaur test
C. Red cell film for haemolysis
D. LDH levels
E. Anti-D titres

Answer: B
A kleihauer test determine if there has been a big fetomaternal bleed and additional anti-D can be
administered. Cell free fetal DNA can be done but is not the most important and readily available
test.

82. Twin-twin transfusion syndrome in a woman with monochorionic twin pregnancies is a


recognized complications.
How high is this risk?
A. 1-5%
B. 10-15%
C. 25-30%
D. 35-40%
E. >50%

Answer: B
The incidence of monochorionic monoamniotic twin is about 1%. The risk of TTTS in MC
pregnancies is 10-15%.

83. Which feature on ultrasound is prognostic of a severe TTTS in a MC pregnancy?


A. Single placental mass
B. Concordant gender
C. Oligohydramnios in one and polyhydramnios in the other
D. Umbilical artery waveforms showing cyclical reduced end diastolic flow
E. Discordant bladder appearances

Answer: E
Based on Quintero classification of TTTS either discordant bladder appearances or signs of
haemodynamic and cardiac compromise are signs of severe TTTS.

84. Following the death of one twin in a monochorionic pair, there is a risk to the surviving
twin.
Which system is most likely to be affected in the surviving twin?
A. Cardiac abnormalities
B. Infection and sepsis
C. Haematological abnormalities
D. Neurologic abnormalities
E. Hydrops

Answer: D
The risk of neurological abnormalities is between 12-18% and delivery should be discussed
based on the gestation age. Such cases should referred to fetal medicine units.

85. The risk of cord entanglement in a monochorionic-monoamniotic twin pregnancy is very


high.
Based on good practice recommendation, what should your advice about delivery be?
A. Caesarean section at 35 weeks
B. Induction of labour at 37 weeks
C. Caesarean section at 32 weeks
D. Scalp electrode for twin 1 early in labour
E. Close monitoring of babies in labour

Answer: C
There is no robust evidence in this area but the Good practice recommendation is to deliver by
caesarean section at 32 weeks after antenatal corticosteroids.

86. In a pregnancy lady at 32 weeks Group B bacteriuria is identified.


Which condition does this predispose her to?
A. Chorioamnionitis
B. Postpartum sepsis
C. Pyelonephritis
D. Neonatal oropharyngitis
E. Neonatal urethritis

Answer: A
Clinicians should offer IAP to women with GBS bacteriuria identified during the current
pregnancy. GBS bacteriuria is associated with a higher risk of chorioamnionitis and neonatal
disease. It is not possible to accurately quantify these increased risks. These women should be
offered IAP.
Women with GBS urinary tract infection (growth of greater than 105 cfu/ml) during pregnancy
should receive appropriate treatment at the time of diagnosis as well as IAP.

87. By using membrane sweeping we can reduce the number of women requiring formal
induction of labour. From systematic review the number needed to treat: 100/15= 6.66 =
7. Hence, seven women need to have a membrane sweep to avoid one formal induction of
labour. In a unit with 3000 deliveries a year where, on average, 20% of women are
induced, there are 600 inductions annually.
If every woman had a membrane sweep, how many inductions could be avoided in a
year?
A. 20-39
B. 40-59
C. 60-79
D. 80-99
E. 100-119

Answer: D
Chapter 2
Postnatal and Neonatal Care

1. For a well infant whose mother had a previous infant with GBS disease, what would be
an appropriate recommendation?
A. Clinical evaluation and observation for 24 hours
B. Blood cultures are not required unless baby is pyrexial.
C. Antibiotics are not required unless baby is culture positive.
D. Antibiotics are indicated if mother had no antibiotic prophylactic.
E. Antibiotics are indicated if mother had antibiotic prophylactic.

Answer: A
The RCOG guideline recommends that clinical evaluation by a neonatologist after birth and
observation for around 24 hours is necessary, or blood cultures need to be obtained and the infant
treated with benzylpenicillin until the culture results are available. It is unclear whether further
action is necessary for the well infant. The risk of GBS disease is unquantified but is probably
significantly increased. The infant should be evaluated clinically soon after birth and observed
for at least 24 hours.

2. A 28-year-old lady delivers her baby vaginally after a significant APH (Antepartum
Haemorrhage).
What are newborn babies of women suffering from APH more likely to suffer with?
A. Hyperbilirubinaemia
B. Hypercalcemia
C. Hypermagnesemia
D. Hyperglycemia
E. Hyperkalemia

Answer: A
These babies are at higher risk of being preterm and hence suffer from early jaundice and
hyperbilirubinaemia.

3. Babies have a higher mortality with cord prolapse.


What is the perinatal mortality rate in the presence of cord prolapse in the UK?
A. 1%
B. 5%
C. 10%
D. 20%
E. 50%

Answer: A
The perinatal mortality is approximately 91/1000 and is mainly related to congenital
malformations, and prematurity of the fetus. Birth asphyxia is another important factor.
4. You are a Specialist Registrar (ST3) and have just been called by the ST1 doctor to help
with a shoulder dystocia. After some initial struggle you manage to deliver the baby with
Apgar score of 5, 7, 9.
What is the most common fetal complication you would anticipated?
A. Clavicle fracture
B. Bell’s palsy
C. Brachial plexus injury
D. Cerebral palsy
E. Humerus fracture

Answer: C
Brachial plexus injury (BPI) is the commonest injury complicating 2-16% of all shoulder
dystocia deliveries. It is the commonest cause for litigation in shoulder dystocia related
deliveries. There can be significant perinatal morbidity and mortality associated with the
condition, even when it is managed appropriately. Maternal morbidity is increased, particularly
the incidence of postpartum haemorrhage (11%) as well as third and fourth degree perineal tear
(3.8%). Their incidences remain unchanged by the number or type of manoeuvres required to
effect delivery.

5. All postnatal women who had pre-eclampsia should have a GP medical review.
What would be the most appropriate timing for review?
A. Before discharge to home
B. 3 days postpartum
C. 10 days postpartum
D. 8 weeks postpartum
E. 12 weeks postpartum

Answer: D

6. Neonatal herpes is a rare condition. Infants present with lesions in various organs.
Which lesion has the best prognosis for the infant?
A. Skin, mouth and eye lesions
B. Disseminated multiple organ lesions
C. Localised CNS lesions
D. Limb lesions
E. Genital lesions

Answer: A
Neonatal herpes is a very rare but serious viral infection with a high morbidity and mortality. It is
classified into three subgroups in the infant depending on the site of infection.
- Disease localized to skin, eye and/or mouth
- Local central nervous system (CNS) disease (encephalitis alone)
- Disseminated infection with multiple organ involvement
Infants who present with symptoms localized to the skin, eye or mouth alone have the best
prognosis and represent approximately 30% of neonatal herpes infections. With appropriate
antiviral treatment, neurological and/or ocular morbidity is less than 2%.
7. In a neonate of a mother with primary herpes who delivered by caesarean section what is
the most appropriate management for the neonate?
A. Intravenous acyclovir prophylactically
B. Swabs from mouth, rectum and oropharynx should be taken
C. Keep in the neonatal unit for 48 hours for observation
D. Close observation
E. No neonatal input as risk of infection is negligible

Answer: A ?
These babies are at low risk of vertically transmitted HSV infection so conservative management
is recommended.
- Liaise with the neonatal team
- Swabs from the neonate are not indicated
- No active treatment is required for the baby
- Normal postnatal care of the baby is advised with a neonatal examination at 24 hours of
age, after which the baby can be discharge form the hospital if well and feeding is
established
- Parents should be educated regarding good hand hygiene and due care to reduce risk of
postnatal infection
- Parents should be advised to seek medical help if they have concerns regarding their
baby. In particular, they should be advised to look for: skin, eye and mucous membrane
lesions, lethargy/irritability, and poor feeding.

8. Besides reducing respiratory distress syndrome is neonates, which other complication do


antenatal steroids reduce?
A. Hypoxic ischaemic encephalopathy
B. Intaventricular harmorrhage
C. Neonatal hypoglycaemia
D. Neonatal acidosis
E. Necrotising enterocolitis

Answer: B
A Cochrane review of 21 studies (3885 woman and 4269 infants) showed that treatment of
women at risk of preterm birth with a single cou8rse of antenatal corticosteroids reduced the risk
of neonatal death by 31% (95% CI 19-42%), RDS by 44% (95% CI 31-57%) and intraventricular
haemorrhage by 46% (95% CI 31-67%). Antenatal corticosteroid use is also associated with a
reduction in necrotizing enterocolitis, respiratory support, intensive care admission and systemic
infections in the first 48 hours of life compared with no treatment or treatment with placebo.

9. Carbimazole crosses the placenta and is occasionally known to cause a scalp defect in the
fetus.
What is this lesion called?
A. Sebaceous nevus
B. Transient bullous dermolysis of the newborn
C. Focal dermal hypoplasia
D. Langerhans cell histiocytosis
E. Aplasia cutis congenital

Answer: E

10. Septic shock cases in the puerperal period cause high numbers of maternal mortality in
the UK.
Which organism is known to be the major pathogen in women who have invasive
infections in the puerperal period?
A. GAS
B. E. coli
C. Streptococcus pneumonia
D. Staphylococcus aureus
E. Methicillin resistant Staphylococcus aureus (MRSA)

Answer: A

11. In severe invasive streptococcal or staphylococcal infection not responding to antibiotics,


what would be the choice of therapy?
A. Platelet transfusion
B. IVIG
C. Heparin infusion
D. Steroid injection
E. Renal dialysis

Answer: B

12. In a woman who develops sepsis on day 3 post caesarean section, what would be the
most common source of infection?
A. Mastitis
B. Urinary tract infection
C. DVT
D. Wound infection
E. Endometritis

Answer: E

13. A woman who had her baby 2 weeks ago has symptoms of depression and delusion. She
has had psychosis after the birth of her last baby.
What are the chances of her developing puerperal psychosis this time?
A. 1:100
B. 1:50
C. 1:25
D. 1:10
E. 1:4
Answer: E
Her risks are as high as 1:4 to 1:2
Chapter 3
Early Pregnancy Complications
1. A 28-year-old woman attends the early pregnancy clinic with bleeding and pain. She is 6
weeks pregnant and very anxious. A transvaginal scan shows an intrauterine gestation sac
measuring 23 x 14 x 12 mm. No fetal pole is seen.
What would you advice?
A. Rescan in 3-5 days
B. Rescan in 10 days
C. Serum β hCG
D. Serum progesterone
E. Await spontaneous miscarriage

Answer: B
If the mean gestational sac diameter is less than 25.0 mm with a transvaginal ultrasound scan and
there is no visible fetal pole, perform a second scan a minimum of 7 days after the first before
making a diagnosis. Further scans may be needed before a diagnosis can be made. If the mean
gestational sac diameter is 25.0 mm or more using a transvaginal ultrasound scan and there is no
visible fetal pole:
- Seek a second opinion on the viability of the pregnancy and/or
- Perform a second scan a minimum of 7 days after the first before making the diagnosis
If there is no visible fetal pole and the mean gestational sac diameter is measured using a
transabdominal ultrasound scan:
- Record the size of the mean gestational sac diameter and
- Perform a second scan a minimum of 14 days after the first before making diagnosis

2. On a transabdominal scan in a 19-year-old lady with bleeding at 7 weeks the sonographer


reports that an intrauterine pregnancy was visualized with a fetal pole measuring 9 mm.
No fetal heart was seen.
What would you advice?
A. Rescan in 14 days as this was a transabdominal scan
B. Confirm miscarriage
C. Rescan in 7 days’ time
D. Perform serial β hCG
E. Insist on a transvaginal scan

Answer: C
If the crown-rump length is less than 7.0 mm with a transvaginal ultrasound scan and there is no
visible heartbeat, perform a second scan a minimum of 7 days after the first before making a
diagnosis. Further scans may be needed before a diagnosis can be made.
If the crown-rump length is 7.0 mm or more with a transvaginal ultrasound scan and there is no
visible heartbeat:
- Seek a second opinion on the viability of the pregnancy and/or
- Perform a second scan a minimum of 7 days after the first before making a diagnosis
If there is no visible heartbeat when the crown-rump length is measured using a transabdominal
ultrasound scan:
- Record the size of the crown-rump length and
- Perform a second scan a minimum of 14 days after the first before making a diagnosis

3. A 39-year-old lady who has suffered with severe endometriosis and has no children is 6
weeks pregnant. A transvaginal scan confirms a 34 mm right sided ectopic pregnancy.
There is a fetal pole but no fetal heart. She is stable. Her β hCG is 3028 iu/l.
What is the safest option of management you would recommend?
A. Laparoscopic salpingectomy
B. Conservative management
C. Systemic methotrexate
D. Repeat scan in 7 days
E. Surgical or medical: give her the choice

Answer: C
In this case scenario an operative procedure is more risky for the mother given severe
endometriosis. Also in her situation conserving her tubes will be the safer option. The safest
option is methotrexate but the patient does have a choice of laparoscopic and medical treatment.
In this question the best answer to the question posed “safest option” is methotrexate.
The NICE guidelines recommend the following: Offer systemic methotrexate as a first-line
treatment to women who are able to return for follow-up and who have all of the following:
- No significant pain
- An unruptured ectopic pregnancy with an adnexal mass smaller than 35 mm with no
visible heartbeat
- A serum β hCG level less than 1500 IU/litre no intrauterine pregnancy (as confirmed on
an ultrasound scan)
Offer surgery where treatment with methotrexate is not acceptable to the woman.
Offer surgery as a first-line treatment to women who are unable to return for follow-up after
methotrexate treatment or who have any of the following:
- An ectopic pregnancy and significant pain
- An ectopic pregnancy with an adnexal mass of 35 mm or larger
- An ectopic pregnancy with a fetal heartbeat visible on an ultrasound scan
- An ectopic pregnancy and a serum β hCG level of 5000 IU/litre or more
Offer the choice of either methotrexate or surgical treatment to women with an ectopic
pregnancy who have a serum β hCG levels of at least 1500 IU/litre and less than 5000 IU/litre,
who are able to return for follow –up and who meet all the following criteria:
- No significant pain
- An unruptured ectopic pregnancy with an adnexal mass smaller than 35 mm with no
visible heartbeat
- No intrauterine pregnancy (as confirmed on an ultrasound scan).
Advise women who choose methotrexate that their chance of needing further intervention is
increased and they may need to be urgently admitted if their condition deteriorates.
For women with ectopic pregnancy who have had methotrexate, take 2 serum β hCG
measurements in the first week (days 4 and 7) after treatment and then 1 serum β hCG
measurement per week until a negative result is obtained. If β hCG levels plateau or rise, reassess
the woman’s condition for further treatment.
4. Anti-D prophylaxis is recommended in all rhesus negative women under 12 weeks of
pregnancy with bleeding in particular situations.
What is a definitive indication of Anti-D under 12 weeks of gestation?
A. Threatened miscarriage
B. Complete miscarriage
C. Medical treatment of ectopic
D. Surgical treatment of ectopic
E. Pregnancy of unknown location

Answer: D
NICE guidance recommends the following:
Offer anti-D rhesus prophylaxis at a dose of 250 IU (50 micrograms) to all rhesus negative
women who have a surgical procedure to manage an ectopic pregnancy or a miscarriage.
Do not offer anti-D rhesus prophylaxis to women who:
- Receive solely medical management for an ectopic pregnancy or miscarriage
- Have a threatened miscarriage
- Have a complete miscarriage
- Have a pregnancy of unknown location
Do not use a Kleihauer test for quantifying feto-maternal haemorrahge.

5. A 19-year-old girl present at 12 weeks of gestation with a missed miscarriage. Although a


fetal pole is seen there are large vascular spaces in the placenta. A molar pregnancy is
suspected.
Based on scan findings what is the most likely diagnosis?
A. Complete molar pregnancy
B. Partial molar pregnancy
C. Placental site trophoblastic tumour
D. Neoplastic nodule
E. Normal pregnancy

Answer: B

6. Epidemiological studies in patients with molar pregnancy suggests that it is more


common in particular group of women.
Which group of women are affected more commonly?
A. Women in their 30s
B. Infertile women
C. Caucasians
D. Smokers
E. Extremes of ages

Answer: E
The risk of molar pregnancy is highest in very young and women above 40. The incidence is
higher in women of Asian ethnicity.

7. How does a complete molar pregnancy differ from a partial?


A. They are usually triploidq
B. They have a better prognosis
C. They arise mostly from duplication of a single sperm
D. They usually contain fetal blood cells
E. The fetus develops when there is a maternal chromosome

Answer: C
Complete moles are always diploid, have a higher risk of malignant potential and never contain a
fetus as there is no maternal chromosome complement.
Molar pregnancies can be subdivided into complete (CM) and partial moles (PM) based on
genetic and histopathological features. Complete moles are diploid and androgenic in origin,
with no evidence of fetal tissue .Complete moles usually (75-80%) arise as a consequence of
duplication of a single sperm following fertilization of an “empty” ovum. Some complete moles
(20-25%) can arise after dispermic fertilization of and ‘empty’ ovum. Partial moles are usually
(90%) triploid in origin, with two sets of paternal haploid genes and one set of maternal haploid
genes. Partial moles occur, in almost all cases, following dispermic fertilization of an ovum. Ten
per cent of partial moles represent tetraploid or mosaic conceptions. In a partial mole, there is
usually evidence of a fetus or fetal red blood cells.

8. A 28-year-old woman who has a copper IUD attends the early pregnancy clinic with a
positive pregnancy test.
What is her risk of an ectopic pregnancy?
A. 1:5
B. 1:20
C. 1:50
D. 1:100
E. 1:200

Answer: B
If a woman becomes pregnant with IUD in situ, the risk of ectopic pregnancy is about 1 in 20,
and she should seek advice to exclude ectopic pregnancy.

9. A 25-year-old woman presents to the A&E department with vomiting. She has
complaints of right sided abdominal pain but no diarrhea. On urinalysis, there are 3+
ketones but no leucocytes.
What is the first test you would consider?
A. FBC
B. Pregnancy test
C. CRP
D. Culture of the MSU
E. Urea and electrolytes

Answer: B
Pregnancy test (usually urine) is the most important test in any woman of reproductive age group
presenting with abdominal pain.
Chapter 4
General Gynaecology
1. While consenting a woman for laparoscopic surgery with no other risk factors the
clinician must explain the frequent risks.
Which one of the following is a frequent risk?
A. Umbilical hernia
B. Wound brusing
C. Damage to blood vessels.
D. Nerve damage
E. Vaginal bleeding

Answer: B
While consenting the clinician performing the surgery must discuss the serious and frequent
risks.
Serious risks include:
- The overall risk of serious complications from diagnostic laparoscopy approximately two
women in every 1000 (2:1000)
Uncommon
- Damage to bowel, bladder, uterus or major blood vessels which would require immediate
repair by laparoscopy or laparotomy (uncommon). However, up to 15% of bowel injuries
might not be diagnosed at the time of laparoscopy
- Failure to gain entry to abdominal cavity and to complete intended procedure
- Hernia at site of entry
- Death; three to eight women in every 100,000 undergoing laparoscopy die as a result of
complications (very rare)
Frequent risks
Frequent risks include:
- Wound bruising
- Shoulder-tip pain
- Wound gaping Wound infection

2. What is the main benefit of flexible outpatient hysteroscopy?


A. Fewer failures
B. Quicker examination
C. Less pain
D. Reduced cost
E. Better image

Answer: C
Apart from less pain, rests are features of rigid hysteroscopes.

3. What is the usual distension medium for outpatient operative hysteroscopy using bipolar
electrosurgery?
A. Carbon dioxide
B. Normal saline
C. Glycine
D. Hartmann’s solution
E. Oxygen

Answer: B

4. A 36-year-old woman attends complaining of chronic pelvic pain. All investigations are
normal. You suspect IBS.
What is the most effective first line of management?
A. Diet modification
B. Laparoscopy
C. Bowel softeners
D. Physiotherapy
E. Mebeverine

Answer: E
Women with cyclical pain should be offered a therapeutic trial using hormonal treatment for a
period of 3-6 months before having a diagnostic laparoscopy.
Women with IBS should be offered a trial with antispasmodics (Recommendation A)
Women with IBS should be encouraged to amend their diet to attempt to control symptoms.
Women should be offered appropriate analgesia to control their pain even adequately therapeutic
manoeuvres are yet to be initiated. If pain is not adequately controlled, consideration should be
given to referral to a pain management team or a specialist pelvic pain clinic.

5. In the study of women with chronic pelvic pain, a systematic review of randomized
controlled trials with a very low risk bias showed that those women with endometriosis
had a significant improvement after resection of the rectal nodules.
Based on the RCOG classification of evidence levels, how would you classify this
evidence?
A. Level 1+
B. Level 1++
C. Level 1
D. Level 2+
E. Level 2++

Answer: B

6. What is the grade of recommendation based on evidence from case reports or expert
opinion classified as?
A. A
B. B
C. C
D. D
E. E

Answer: D
7. Symptoms of severe dyschezia in a patient known to have endometriosis is common.
Which type of lesion is this suggestive of?
A. Uterovesical fold lesion
B. Widespread large bowel lesion
C. Disseminated lesion to the diaphragm
D. Deep rectovaginal septum lesion
E. Lesion of lateral pelvic walls

Answer: D
The symptoms of endometriosis depend on the location of the disease. Deep endometriosis of the
posterior pelvis is associated with increased severity of dyschezia, in comparison to women with
pelvic endometriosis without posterior deep endometriosis. Deep endometriosis of the
rectovaginal septum is associated with the most severe forms of dyschezia and dyspareunia

8. A 32-year-old woman underwent a transvaginal ultrasound for right sided pelvic pain.
The scan suggests a right ovarian cyst 53 x 43 x 62 mm with some ground glass
echogenicity, three compartments and no papillary structures?
What is the likely diagnosis?
A. Dermoid cyst
B. Endometrioma
C. Mixed Serous cystadenoma
D. Adhesions
E. Hydrosalpinx

Answer: B
The GDG (Guideline Development Group) recommends that clinicians base the diagnosis of
ovarian endometrium in premenopausal women on the following ultrasound characteristics:
ground glass echogenicity and one to four compartments and no papillary structures with
detectable blood flow.

9. In the diagnosis of rectal endometriosis several investigations are used. Which


investigation is most valuable?
A. Transvaginal ultrasound
B. MRI
C. CT scan
D. CA 125
E. 3-D ultrasound

Answer: A

10. A woman presents with pain from rectovaginal endometriosis refractory to medical and
surgical treatment.
What might another recommended option be in her management?
A. Antidepressants
B. Nerve ablation (LUNA)
C. Gabapentin
D. Aromatase inhibitors
E. Progestagen progestogen

Answer: D
In women with pain from rectovaginal endometriosis refractory to other medical or surgical
treatment, clinicians can consider prescribing aromatase inhibitors in combination with oral
contraceptive pills, progestagens, or GnRH analogues, as they reduce endometriosis associated
pain. As aromatase inhibitors also can induce ovulation induction, contraception should be used
with aromatase inhibitors.

11. Secondary prevention of endometriosis associated dysmenorrhea in patients undergoing


surgery for endometriosis should be considered.
What is a suitable recommendation?
A. Amitriptyline
B. Gabapentin
C. GnRH analogues
D. Levonorgestrel IUS
E. Aromatase inhibitors

Answer: D
In women operated on for endometriosis, clinicians are recommended to prescribe postoperative
use of a levonorgestrel-releasing intrauterine system (LNG-IUS) or a combined hormonal
contraceptive for at least 18-24 months, as one of the options for the secondary prevention of
endometriosis-associated dysmenorrhea, but not for non-menstrual pelvic pain or dyspareunia.

12. A woman with ASRM stage 1/2 endometriosis with endometriosis is undergoing
IVF/ICSI.
What is a recommended drug during oocyte retrieval?
A. General anaesthesia
B. NSAIDs
C. Antibiotics
D. LMWH
E. Progesterone

Answer: C
In women with endometrioma, clinician may use antibiotic prophylaxis at the time of oocyte
retrieval, although the risk of ovarian abscess following follicle aspiration is low.

13. A woman with known endometriosis is very concerned about the risk of cancer.
Based on current evidence what is the information you would provide her?
A. Endometriosis increases the risk of cancer by 10%
B. Endometriotic lesions can directly cause cancer in 15% of cases
C. Non-Hodgkin’s lymphoma is more common in women with endometriosis
D. Endometrial cancers are less common in women with endometriosis
E. Bowel cancers are less common in women with endometriosis
Answer: C
The GDG recommends that clinicians inform women with endometriosis requesting information
on their risk of developing cancer that
(1) There is no evidence endometriosis cause cancer
(2) There is no increase in overall incidence of cancer in women with endometriosis, and
(3) Some cancers (ovarian cancer and non-Hodgkin’s lymphoma) are slightly more common
in women with endometriosis.

14. A 52-year-old woman is started on combined hormone replacement therapy for


vasomotor symptoms after careful counseling about the risks of thromboembolism.
When does she have the highest risk of venous thromboembolism?
A. After 5 years of use
B. After 4 years of use
C. After 3 years of use
D. In the second year of use
E. In the first year of use

Answer: E
The risk of VTE is highest in the 1st year of use, with no evidence of continuing risk on stopping
HRT.

15. The use of combination HRT changes some of the haemostatic systems in the body to
produce a procoagulant state.
What change in the haemostatic system with HRT predisposes to a procoagulant state?
A. Slight increase in fibrinogen
B. Consistent fall in plasminogen activator inhibitor-1 levels
C. Increase in protein S
D. Increase in Factor VII
E. Decrease in plasminogen activation factor

Answer: B
With HRT there is a reduction in fibrinogen and a reasonably consistent decrease in plasminogen
activator inhibitor-1 levels have been observed, suggesting an overall enhancement of
fibrinolytic potential in those taking HRT. In addition, combination HRT may reduce factor VII
levels. Oral HRT also reduces plasma levels of the natural anticoagulant protein S levels but has
minimal effect/slight increase on plasminogen activation factor.

16. Recently published figures suggests that there is a background incidence of VTE in
women of reproductive age group.
What is the true incidence of VTE in a year?
A. 1:1000
B. 2:10,000
C. 3:10,0000
D. 5:1000
E. 7:10,000
Answer: B
The term venous thromboembolism (VTE) includes deep vein thrombosis (DVT), pulmonary
embolism, and cerebral venous sinus thrombosis. Most studies of venous thrombosis and
hormonal contraceptive use relate to DVT and pulmonary embolism. The true background
incidence of VTE in women of reproductive age is difficult to quantify but recently published
figures suggest it is in the range of 2 per 10,000 women in 1 year.

17. What is the risk of VTE in women using combined hormonal contraception?
A. 1 per 10,000
B. 2-4 per 10,000
C. 5-12 per 10,000
D. 15-20 per 10,000
E. 25-30 per 10,000

Answer: C
The risk of VTE with use of CHC was therefore reported to range from 5-12 per 10,000 women
years, compared with 2 per 10,000 nonusers. However the EMA noted that the benefits of CHC
use generally outweighed the risk of venous thrombosis, which is low overall and is lower than
the VTE risk associated with pregnancy and the postpartum period (29 per 10,000 woman-years
and 300-400 per 10,000 woman-years, respectively). Despite evidence of more favourable
effects on lipid profiles and carbohydrate metabolism, there is no evidence to suggest that the
newer, less androgenic progestogens are any safer in terms of arterial thrombosis risk than older
progestogens.

18. When is the risk of VTE highest after starting combined hormonal contraception?
A. In the first one month
B. In the first 4 months
C. In the first year
D. In the first 3 years
E. Remains the same while used

Answer: B
The risk of VTE is highest in the four months following initiation of CHC or when restarting
after a break of at least one month. The risk then reduces over the next year and remains stable
thereafter. Although the risk is high in the first few months of CHC use and then falls, it remains
higher than in non-users.

19. The progestogens are classified into generations based on the time they were first
marketed.
What is a commonly used fourth generation progestogen?
A. Desogestrel
B. Gestodene
C. Co-cyprindiol
D. Dienogest
E. Norgestimate
Answer: D
Progestogens are often grouped according to the time they were first marketed as constituents of
COCs and may be referred to by ‘generation’.
- First generation are Norethisterone, norethisterone acetate
- Second generation Levonorgestrel
- Third generation Desogestrel, gestodene, norgestimate
- Fourth generation Drospirenone, dienogest, nomegestrol acetate

20. A 32-year-old woman comes with complaint of heavy menstrual bleeding for 8 months.
No physical abnormality was noted on examination. She feels very fatigued.
What investigation would you consider first?
A. Full blood count
B. Ferritin
C. Thyroid profile
D. Day 2 hormone profile
E. Coagulation profile

Answer: A
FBC should be done and ferritin is not indicated as iron deficiency is the likely cause. Thyroid
profile is not indicated unless other symptoms present. Coagulation profile is indicated for
primary menorrhagia and not for secondary.

21. A 28-year-old woman gives a four month history of heavy regular periods with flooding.
She is fit and has no medical problems. She is worried about having to miss work. She is
currently using condoms for contraception.
What would be the most suitable first option of treatment in her case?
A. Tranexamic acid and mefenamic acid
B. Combined contraceptive pill
C. LNG-IUS
D. Endometrial ablation
E. Copper IUCD

Answer: C
If history and investigations indicate that pharmaceutical treatment is appropriate and either
hormonal or non-hormonal treatments are acceptable, treatments should be considered in the
following order.
1. Levonorgestrel-releasing intrauterine system (LNG-IUS) provided long-term (at least 12
months) use is anticipated.
2. Tranexamic acid or non-steroidal anti-inflammatory drugs (NSAIDs) or combined oral
contraceptives (COCs)
3. Norethisterone (15 mg) daily from days 5 to 26 of the menstrual cycle, or injected long-acting
progestogens.
If hormonal treatments are not acceptable to the woman, then either tranexamic acid or NSAIDs
can be used.
22. A 42-year-old lady who has completed her family attends your clinic with complaints of
heavy menstrual period. She is found to have multiple fibroids over 3 cm in her uterus.
She also had some pressure symptoms.
What is the first option of treatment you would offer?
A. Tranexamic acid and mefenamic acid
B. UAE
C. Myomectomy
D. Hysterectomy
E. Levonorgestrel IUS

Answer: B
For women with large fibroids and HMB, and other significant symptoms such as dysmenorrhea
or pressure symptoms, referral for consideration of surgery or uterine artery embolization (UAE)
as first-line treatment can be recommended.

23. While counseling a woman who is going to have a copper IUD inserted you must explain
about the risk of uterine perforation.
What is this risk during the insertion?
A. 1:50
B. 1:100
C. 1:200
D. 1:500
E. 1:1000

Answer: E
The risk of uterine perforation at the time of IUD insertion is very low (less than 1 in 1000).

24. A woman who has a copper IUD attends the gynaecology clinic referred by her GP.
During a cervical smear test taken by the GP there is evidence of Actinomyces-like
organisms.
What is the next step in her management?
A. Remove the IUD
B. Take triple swabs
C. Repeat smear test
D. Request a scan to check for adnexal masses
E. Commence on Penicillin

Answer: C
It is not necessary to remove the IUD without signs of pelvic infection. The first step would
entail triple swabs.
Chapter 5
Colposcopy and Gynaecological Oncology
1. A 60-year-old nulliparous woman is incidentally found to have an ovarian cyst measuring
5 cm on scan.
What should be the next step in her management?
A. Repeat scan in 6 months
B. CA125
C. Surgery by general gynaecologist
D. Laparoscopic surgery
E. MDT discussion

Answer: B
CA 125 is required to calculate RMI before further decision can be made by the MDT team.

2. A 60-year-old woman is incidentally found to have a right sided ovarian cyst measuring
5.1 x 5.8 cm. RMI is calculated to be 25.
What is the appropriate treatment?
A. Ovarian cystectomy at a cancer centre
B. Laparoscopic ovarian cystectomy
C. Laparoscopic bilateral oophorectomy
D. Conservative management with serial scans
E. Discharge

Answer: C
RMI <25 is for conservative measurement. Laparoscopic surgery is appropriate. Cystectomy
alone is suboptimal management in post-menopausal women.

3. What kind of personal or family history would you expect to find in a woman with lichen
sclerosus?
A. Cancers
B. Abnormal cervical cytology
C. Cystic fibrosis
D. Autoimmune diseases
E. Gluten intolerance

Answer: D
Women with lichen sclerosus and erosive lichen planus are at increased risk of a personal or
family history of autoimmune disorders. Circulating autoantibodies are more frequent in women
with erosive lichen planus. The most common autoimmune conditions in women with lichen
sclerosus are thyroid disorders, alopecia areata, pernicious anaemia, type1 diabetes and vitiligo.
The reported prevalence of autoimmune conditions in first-degree relatives is around 30%.

4. There is a known risk of VIN progressing to cancers.


What is the long-term risk?
A. 1%
B. 5-7%
C. 10-20%
D. 20-30%
E. 40-60%

Answer: E
VIN has a well-recognised risk of developing into a squamous cell cancer. Several cohort studies
have found cancer to develop within 8 years of diagnosis; the reported risk of progression to
cancer varies widely, but appears to be in the order of 40-60%. The risk is higher in untreated
women. The risk of unrecognized invasion means a low threshold should be set for biopsy.
Women treated surgically for VIN still have a residual risk of developing invasive cancer in the
order of 4%.

5. A woman is diagnosed with VIN on a skin biopsy of her vulva.


What kind of lesion could this develop into long term?
A. Squamous cell carcinoma
B. Adenocarcinoma
C. Basal cell carcinoma
D. Kaposi’s sarcoma
E. T cell lymphoma

Answer: A
VIN has a well recognized risk of developing into a squamous cell cancer.

6. It is recommended that women who have been treated for VIN should be reviewed in a
specialist vulval clinic.
What is the recommended interval for review?
A. 3 monthly intervals
B. 6 monthly intervals
C. 1 yearly intervals
D. 2 yearly intervals
E. 5 yearly intervals

Answer: C

7. Which type of HPV is strongly related to cervical cancer?


A. HPV type 16
B. HPV type 2
C. HPV type 11
D. HPV type 6
E. HPV type 21

Answer: A
HPV type 16 and 18 cause 70% of cervical cancers and the HPV vaccine currently protect
against these two strains of virus but also offers some cross protection against 6 and 11 types
which mainly cause anogenital warts.
8. Certain factors are protective for the occurrence of endometrial cancer.
What in the personal history is a protective factor for endometrial cancer?
A. Early menarche
B. Obesity
C. Early menopause
D. Nulliparity
E. Old age

Answer: C
Several risk factors such as obesity, tamoxifen use, increasing age, hypertension, diabetes and
unopposed use of exogenous oestrogens are strongly associated with increased risk of type-1
endometrial cancer. Early menarche and late menopause have also been implicated due to
prolonged oestrogen stimulation of the endometrium. Nulliparity alone an isolated risk factor
does not appear to increase the risk of endometrial cancer. Although due to the more of
anovulatory cycles there may be an association in women with infertility. Hereditary nonn-
polyposis colorectal cancer is a significant but rare risk factor. Those with an affected family
member with a history of hereditary non-polyposis colorectal acncer have a theoretical lifetime
risk of 50% of getting endometrial cancer.

9. A 28-year-old woman smokes 20 cigarettes per day. Her cervical smear result has been
reported as inadequate.
What would you recommend as the next course of action?
A. Repeat smear in three months
B. Perform a HPV test
C. Repeat smear as soon as possible
D. Organise a colposcopy
E. Routine recall in three years

Answer: A
Inadequate smears need repeat smear in three months.

10. A 26-year-old woman presents with recurrent chlamydia infection. Her cervical smear
result has shown low grade dyskaryosis.
What is the best course of action?
A. Repeat smear in three months
B. Perform HPV test
C. Repeat smear as soon as possible
D. Organize a colposcopy
E. Routine recall in three years

Answer: B
A HPV test should be performed. HPV negative patients can go back to regular three yearly
recall.

11. A 32-year-old woman is a heavy smoker. Her cervical smear result was reported as low
grade dyskaryosis. HPV test is inadequate.
What is the best course of action in this situation?
A. Repeat smear in three months
B. Repeat HPV test
C. Repeat smear and HPV test as soon as possible
D. Organize a colposcopy
E. Routine recall in three years

Answer: D

12. A 28-year-old woman smokes 20 cigarettes per day. Her cervical smear was reported as
abnormal. Colposcopy revealed CIN 3 which was treated.
What is the best course of action for follow-up?
A. Repeat smear in three months
B. Repeat smear and HPV test in six months
C. Repeat smear and HPV test as soon as possible
D. Organize a colposcopy
E. Regular recall in three years

Answer: B

13. A 28-year-old woman with a history of repeated vaginal infections and multiple sexual
partners attends the GUM clinic. Her cervical smear result is reported as high grade
dyskaryosis.
What would you advise next?
A. Repeat smear in three months
B. Do HPV test to support diagnosis
C. Repeat smear as soon as possible
D. Organize colposcopy
E. Routine recall in three years

Answer: D
High grade dyskaryosis does not require HPV testing and can be referred directly for
colposcopy.

14. A 50-year-old woman presents with history of recurrent postcoital bleeding.


What is her risk of cervical cancer based on epidemiology?
A. 1:1000
B. 1:2500
C. 1:5000
D. 1:10,000
E. 1:20,000

Answer: B
Few facts about cervical cancer
- Point prevalence of postcoital bleeding in the community is 0.7-9%
- Point prevalence of postcoital bleeding in women with cervical cancer is 0.7-39%
- Risk of cervical cancer in a woman (20-24 years) presenting with postcoital bleeding is
1:44,000
- Risk of cervical cancer in a woman (45-54 years) presenting with postcoital bleeding is
1:2400

15. A woman who is 35 years of age presents with a history of irregular bleeding for 3
months. Her periods are heavy. She has had 3 children by caesarean section. She is using
condoms form contraception and pregnancy test is negative.
What is the most likely cause of her intermenstrual bleeding?
A. Vaginal cancer
B. Submucous myomas
C. Vaginitis
D. Vaginal adenoma
E. Condyloma acuminate

Answer: B
All the above are causes of irregular vaginal bleeding but if the periods are already heavy in a
35-year-old the commonest diagnosis would be submucous polyps/myomas.
The causes are
- Hormonal imbalance POP, OCP (especially low dose)
- Vaginitis
- Submucosal Myomas
- Condyloma accuminata
- Cancer cervix
- Vaginal adenosis
- Vaginal cancer
- Pharmacological: anti-psychotics, anti-epileptics, corticosteroids and anti-coagulants in
women taking OCP

16. A 22-year-old woman presents with persistent post coital bleeding for 6 weeks. You see a
large ectropion on speculum. You take a smear test.
What should be the next course of action?
A. Refer to GUM clinic
B. Take triple swabs
C. Advise on condoms as additional use
D. Treat the ectropion with silver nitrate
E. Refer for colposcopy

Answer: B
The first course of action in young women with persistent postcoital bleeding is to rule out
chlamydia and gonorrhea. Smear test may be done, refer to GUM medicine but also refer for
colposcopy.
The indication for colposcopy are the following:
- Persistent postcoital bleeding for >4 weeks in women >35 years without any unusual
featurs
- In women <24 years 6-8 weeks, referral to GU Medicine and colposcopy
- Friable cervix (aim for biopsy and ablative treatment if needed)

17. Pipelle endometrial biopsy is done routinely in postmenopausal women to diagnose


endometrial cancer.
What is the likelihood that an adequate sample will diagnose endometrial cancer?
A. 40-50%
B. 55-65%
C. 65-75%
D. 75-85%
E. >85%

Answer: E
An adequate endometrial Pipelle biopsy is:
- Simple office procedure
- Only considered in high risk premenopausal women (level II evidence)
- Adequate sample is obtained in more than 85% of the time and detects 87% to 96% of
endometrial carcinoma

18. In a 45-year-old woman with irregular vaginal bleeding, an endometrial pipelle biopsy
yielded a histologically insufficient sample. Her BMI is 55. A scan shows endometrial
thickness of 7 mm on day 9 of her cycle.
What would be your next step in management?
A. Reassure and see again if further bleeding
B. Arrange hysteroscopy under general anaesthetic
C. Arrange outpatient hysteroscopy
D. Arrange a repeat endometrial biopsy
E. Repeat scan in 6 months

Answer: A
In women with irregular uterine bleeding no further investigation is required if insufficient
sample is obtained for histological diagnosis and outpatient hysteroscopy is not possible due to
BMI, other reasons such as patient choice, you can wait and watch.
- If there is no further bleeding
- If the endometrial thickness is <12 mm in premenopausal and < 5 mm in postmenopausal
woman
Chapter 6
Fertility
1. A 26-year-old woman has been trying to conceive for the last 2 years. She is morbidly
obese and is diagnosed with PCOS. She is advised to undergo laparoscopic gastric
banding procedure to lose weight and expedite conception.
What would be your advice regarding conception after surgery?
A. To try and conceive immediately after surgery
B. To wait till skin stitches have been removed and she is comfortable
C. Use contraception till after 6 months of surgery
D. Wait till optimum weight has been achieved
E. Use contraception till after 12 months of surgery

Answer: E
Conception should be avoided in the immediate phase of rapid weight loss after surgery as
vitamin and essential mineral malabsorption can lead to high pregnancy loss and fetal
complications. Pregnancy can happen unexpectedly especially in younger women so it is
advisable to use contraception till the phase of irregular absorption is over and there are no
surgery related complications like slippage of the band. Optimum weight may not be achieved
till many years after surgery and conception should not be deferred indefinitely.

2. A 32-year-old nulliparous woman has been diagnosed with two fibroids 8 x 7 cm and 7 x
6 cm and 7 x 6 cm. She would like to know about the benefits of uterine artery
embolization.
What is the main benefit of UAE in her case?
A. Shorter hospital stay
B. Decreased likelihood of surgery
C. Improved quality of life
D. Better fertility outcome
E. Better satisfaction rate

Answer: A
UAE has a similar satisfaction rate as myomectomy or hysterectomy. The advantage is shorter
hospital stay and early return to activities. There is an increased likelihood of surgical
intervention at 2-5 years. Fertility outcomes may be slightly better with myomectomy based on
poor evidence. Studies are ongoing in this area.

3. A 32-year-old nulliparous woman has been diagnosed with two fibroids 8 x 7 cm and 7 x
6 cm. She is contemplating about the option of UAE. There are some contraindications to
this procedure.
What is the main contraindication?
A. Desire for future pregnancy
B. History of pelvic infection
C. Symptomatic fibroid
D. Pregnancy
E. Multiple abdominal surgeries
Answer: D
Pregnancy must be ruled out in a young woman contemplating UAE

4. A 30-year-old woman has primary infertility. She is fit and well and has been unable to
conceive despite regular unprotected intercourse for the past 1 year. Investigations
indicate normal semen analysis, patent fallopian tubes and evidence of ovulation.
What should you advise?
A. Refer for IVF
B. Ovulation induction with clomiphene citrate
C. To continue to try to conceive naturally for further 1 year
D. Offer IUI
E. Have timed intercourse in ovulatory period

Answer: C
Diagnosis is unexplained infertility. NICE guidance (fertility: assessment and treatment for
people with infertility problems) advises referral for IVF after 2 years of unexplained infertility
and clomiphene is not recommended in unexplained infertility.

5. A 37-year-old primigravida who is fit and well has been unable to conceive despite
regular unprotected intercourse for the past 6 months. She attends her GP’s surgery for
advice and is referred to the secondary care.
What would you advise?
A. Referral for IVF
B. Ovulation induction with clomiphene citrate
C. Continue to try to conceive naturally for further 12 months
D. Offer IUI
E. Offer further investigations for infertility

Answer: E
Since she is more than 36 years old, referral earlier than one year is recommended.

6. A 39-year-old lady with primary infertility has been trying to conceive for the past 6
months and is being investigated. There are concerns about her ovarian reserve.
Which test will you offer in you clinic to check for ovarian reserve?
A. Ovarian volume
B. Ovarian blood flow
C. Inhibin B
D. Oestradiol (E2)
E. FSH

Answer: E
Rests of the tests are not routinely recommended

7. A 29-year-old lady has primary infertility. She attends the fertility clinic. Her cycles are
very irregular for the last 10 years.
What would be the most reliable test to check for ovulation in this patient?
A. Basal body temperature
B. Antral follicle count
C. Serum progesterone
D. FSH
E. Inhibin B

Answer: C
Despite her cycles being irregular serum progesterone remains the standard test for evidence of
ovulation, although it may need to be repeated to get an accurate result.
Chapter 7
Urogynaecology
1. Botulinum toxin injection has been used recently for the management of overactive
bladder. What is the recommended site of administration?
A. At the trigone of the bladder
B. Around the internal urethral sphincter
C. Into the detrusor muscle
D. Cystoscopically into the bladder lumen
E. Around the external urethral meatus

Answer: C
Botulinum toxin is administered via a cystoscope and injected into the detrusor from within,
usually at 20-30 sites across the dome of the bladder, and usually sparing the trigone. It can be
administered under local or general anaesthesia via rigid or flexible cystoscopy. Flexible
cystoscopy and local anaesthesia is quick, easy and carries few risks.

2. In the treatment of overactive bladder, several options are available.


Which option should be used as a second line of treatment after anticholinergic
treatment?
A. Botulinum toxin
B. Reduce caffeine drinks
C. Bladder drills
D. Pelvic floor exercises
E. Sacral nerve stimulation

Answer: E
Sacral nerve stimulation should be considered as second line after anti-cholinergic treatment
(combination of two drugs) has failed.

3. You are assessing for urethral competence in a woman presenting with symptoms of
mixed urinary incontinence.
What would be the most appropriate and an useful test to consider?
A. Bladder diary
B. Pad tests
C. Multichannel cystometry
D. Fluid-Bridge test
E. Q-tip test

Answer: A
Do not use the Q-tip, Bonney, Marshall and Fluid-Bridge tests in the assessment of women with
UI.

4. Bladder training as part of behavioural therapy is the first line management for mixed
urinary incontinence.
What is the minimum period this should be offered for?
A. 4 weeks
B. 6 weeks
C. 8 weeks
D. 10 weeks
E. 12 weeks

Answer: B
Bladder training lasting for a minimum of 6 weeks should be offered as first-line treatment to
women with urgency or mixed UI.

5. What is the drug of choice in the treatment of overactive bladder in frail older women?
A. Flavoxalate
B. Imipramine
C. Propantheline
D. Oxybutynin
E. Darefenacin

Answer: E
Flavoxate, propantheline and imipramine should not be used in women for the treatment of OAB
in women. Oxybutynin (immediate release) should not be given to frail older women. The best
choice of drugs are Darefenacin and tolterodine.

6. What is the most common side effect of Mirabegron?


A. Low platelets
B. Mouth ulcers
C. Haemolytic anaemia
D. Eyelid and lip oedema
E. Renal dysfunction

Answer: D
There are several adverse reactions for mirabegron: urinary tract infection, tachycardia, vaginal
infection, cystitis, palpitation, atrial fibrillation, dyspepsia, gastritis, urticarial, rash, rash
macular, rash popular, pruritus, joint swelling, vulvovaginal pruritis, increased blood pressure,
increased gamma-glutanyl transpeptidase, increased aspartate aminotransferase, increased
alanine aminotransferase, eyelid oedema, lip oedema, leukocytoclastic vasculitis and purpura
(rash).

7. While taking history in a woman presenting with mixed urinary incontinence you
determine that stress incontinence is the main symptom.
What is your first line of management?
A. Botox injection
B. Urodynamic testing
C. Reducing caffeine intake
D. Cystoscopy
E. Trial of Mirabegron

Answer: C
Conservative measures should be the first line of management. Urodynamics should not be used
as first line before measures such as reducing fluids, caffeine, weight loss and pelvic floor
exercises are introduced.

8. In a woman with a posthysterectomy vault prolapse, you consider abdominal


sacrocolpopexy and sacrospinous fixation.
What is the main drawback of abdominal sacrocolpopexy over sacrospinous fixation?
A. Similar operating time
B. More overall morbidity
C. More bladder injuries
D. More vault haetomas
E. More vaginal pain

Answer: B
Chapter 8
Sample Question Paper
1. Bacterial vaginosis infection can cause a smelly vaginal discharge. A patient is keen to
know more about this.
What would you advise about the nature of this infection?
A. It is a sexually transmitted infection
B. It is rare in women in the same sex relationship
C. It increases the risks of acquiring other STIs
D. Most women will experience vaginal soreness and itching
E. It tends to recur and is difficult to treat

Answer: C
Bacterial vaginosis (BV) does not affect men and is not a sexually transmitted disease. It is
common in women in the same sex relationship. It increases the risks of acquiring other STIs and
therefore testing for Chlamydia, gonorrhea, trichomonas, syphilis and HIV should be offered in
women diagnosed with BV. It is asymptomatic in 50% of women and rarely causes soreness and
itching. It is easily treated with metronidazole or clindamycin.

2. Thrichomonas vaginal is a common curable sexually transmitted infection.


What is the organism causing this?
A. Anaerobic bacteria
B. Gram-negative bacteria
C. Protozoon
D. Atypical fungus
E. Diploid fungus

Answer: C
Trichomonas vaginalis is caused by a protozoan infestation.

3. What percentage of women with chlamydia will develop pelvic inflammatory disease?
A. 3-5%
B. 10-15%
C. 20-25%
D. 30-40%
E. 45-50%

Answer: B

4. What proportion cervical cancers are caused by HPV type 16 and 18?
A. 20% of cervical cancers
B. 40% of cervical cancers
C. 55% of cervical cancers
D. 70% of cervical cancers
E. 90% of cervical cancers

Answer: D
5. A mother presents with known Marfan’s syndrome at 32 weeks of pregnancy. There is no
other indication for caesarean section. Patient is keen to have a vaginal delivery.
What finding on the echocardiogram would be an absolute indication for a caesarean
section?
A. Mildly dilated left ventricle
B. Aortic root dilatation of 2 cm
C. Aortic root dilatation of 4 cm
D. Current β-blocker treatment
E. Previous aortic root replacement

Answer: C
Aortic root dilatation secondary to cystic medial necrosis occurs in association with Marfan’s
syndrome, familial thoracic aneurysm, bicuspid aortic valve, and repaired tetralogy of Fallot. It
confers a risk of type A dissection, most commonly in the third trimester or immediately post
partum – the time of greatest haemodynamic sheer stress to the aortic wall. Most of the literature
on Marfan’s syndrome suggests that the overall mortality of pregnancy is 1%, with the risk of
aortic dissection increasing significantly if the aortic root diameter is greater than 4 cm or
progressively dilating. These women should be advised to consider aortic root replacement
before conception. This 4 cm cut off can be extrapolated to non-Marfan’s aortic root dilatation
and echocardiograms should be performed before conception and every four to six weeks during
pregnancy. It is probably sensible for all, and essential for the higher risk group, to be fully β
blocked throughout pregnancy. Low risk cases may have a normal delivery with an assisted
second stage but most specialist units recommend caesarean section for higher risk women.

6. A woman attending for preconception counseling has a history of congenital atrial septal
defect. She would like to know the risks of her baby developing congenital heart disease.
What is her risk?
A. 1%
B. 3-6%
C. 8-10%
D. 15%
E. 20%

Answer: B

7. A G2P1 woman attends a preconception clinic. Her first baby was born with
anencephaly. What dose of folic acid would you advise?
A. 400 micrograms daily
B. 500 micrograms daily
C. 600 micrograms daily
D. 700 micrograms daily
E. None of the above

Answer: B
8. Clostridium difficile is a common hospital acquired infection.
Which drug is used in recurrent C difficile infection to reduce recurrence rates
significantly?
A. Vancomycin
B. Metronidazole
C. Rifaximin
D. Meropenem
E. Fidaxomicin

Answer: E
A systematic review published in 2011 concluded that no antimicrobial agent is clearly superior
for the initial cure of CDI (C Difficile Infection), but that recurrence is less frequent with
fidaxomicin than with vancomycin. SMC (Scottish Medicines Collegiate, 2012) concluded that
Fidaxomicin is appropriate for the treatment of adults with a first episode of CDI recurrence, on
the advice of local microbiologists or specialists in infectious diseases. NICE reviewed the
strengths and weaknesses of the relevant evidence regarding Fidaxomicin, but its summary does
not represent formal NICE guidance. NICE concluded that Fidaxomicin may have advantages in
reducing the rate of recurrence, and that local decision makers should take into account the
potential benefits alongside the medical need, the risks of treatment, and the relatively high cost
of the antibiotic in comparison with other CDI treatment options.

9. A postnatal lady on day 10 post caesarean section is found to have sepsis. The surgeons
suspect necrotising fasciitis of lower abdominal wall.
What is the most common organism that causes this?
A. Klebsiella
B. Clostridium
C. Escherichia coli
D. Staphylococcus aureus
E. Group A streptococci

Answer: E

10. Which one of the serological tests is routinely done at antenatal booking?
A. Hepatitis A
B. Hepatitis B
C. Influenza
D. CMV
E. Varicella

Answer: B

11. There is a risk of premature menopause in some women having hysterectomy at a


younger age. The reason is that part of the blood supply to the ovary comes from the
uterine artery but majority from the ovarian artery.
Ovarian artery is a branch of which artery?
A. Renal artery
B. Anterior branch of internal iliac artery
C. Inferior epigastric artery
D. Abdominal aorta
E. External iliac artery

Answer: D
The ovarian artery arises from the abdominal aorta just below the renal artery.

12. Breastfeeding immediately after delivery of a baby can reduce the risk of bleeding by
causing uterine contraction. Which hormone is released to cause this?
A. VEGF
B. Oxytocin
C. Prolactin
D. Prostacyclin
E. Prostaglandin F2α

Answer: B

13. A 14-year-old girl is diagnosed with an early stage malignant germ cell tumour. She
undergoes surgery and is completely treated.
Which tumour marker is used for surveillance?
A. CA125
B. CEA
C. AFP
D. CA199
E. LDH

Answer: C

14. A 35-year-old lady with a family history of premature menopause attends your clinic in
an anxious state. She has no children and although has had long cycles, she has for the
first time had an amenorrhoea of 11 months. She is otherwise fit and well. Her estradiol
levels are normal. What is the most likely provisional diagnosis?
A. Hyperprolactinaemia
B. Premature ovarian failure
C. Kallmann’s syndrome
D. Hypothyroidism
E. Polycystic ovarian syndrome

Answer: E

15. A couple present with a history of primary subfertility for 2 years. Semen analysis is
normal. Day 21 progesterone suggests ovulation. Female partner has high levels of
chlamydia antibodies (CAT).
What would be the most appropriate test for tubal patency?
A. Hysterosalpingogram
B. Laparoscopy and dye test
C. Saline hysterosonography
D. HyCoSY
E. Hysteroscopy and selective salpingography

Answer: B
For those with a history of Chlamydia and positive Chlamydia antibodies, laparoscopy and dye
test remains the gold standard for tubal patency test.

16. A 28-year-old lady has undergone a left laparoscopic ovarian cystectomy for an 8 cm
endometrioma. She was readmitted on day 3 of surgery with symptoms of increasing
abdominal pain and fever. There is no abdominal distension but she is markedly tender in
the left loin.
What is the most likely provisional diagnosis?
A. Acute pyelonephritis
B. Bladder injury
C. Bowel injury
D. Ureteric injury
E. Pelvic haematoma

Answer: D

17. A 24-year-old patient presents with secondary amenorrhoea. She has a recent history of
anorexia.
What is the most likely diagnosis?
A. Premature ovarian failure
B. Hypo-prolactinemia
C. PCOS
D. Hypo-gonadotrophic hypogonadism
E. Hyperthyroidism

Answer: D

18. During labour, occipito-posterior position of the fetal head is favoured in which type of
pelvis?
A. Android
B. Platypeloid
C. Anthropoid
D. Gynecoid
E. Mixed type

Answer: A
Occipito-posterior is favoured by both android and anthropoid type of pelvis but is more
common in android.

19. A patient is found to have hyperthyroidism in pregnancy.


Which condition causes 95% of cases of hyperthyroidism in pregnancy
A. Thyroiditis
B. Multinodular goitre
C. Toxic adenoma
D. Graves’ disease
E. Subacute thyroiditis

Answer: D

20. Thyroid nodules are rare occurring in about 1% of women in reproductive age group.
However when discovered in a woman during pregnancy have an increased risk of
malignancy?
What is this risk?
A. 10%
B. 20%
C. 30%
D. 40%
E. 50%

Answer: E

21. A pregnant woman with a family history of hypothyroidism in her mother and sister is
found to be positive for thyroid peroxidase antibodies. She is likely to develop
postpartum thyroiditis.
When is her risk highest?
A. Just after delivery
B. 48-72 hours postpartum
C. 3-4 weeks postpartum
D. 3-4 months postpartum
E. After 6 months postpartum

Answer: D

22. A 20-year-old woman in her first pregnancy presents to the antenatal clinic at 14 weeks
gestation with a history of known hypothyroidism. She is currently euthyroid and is
taking 100 µg/day of thyroxine.
What is an important factor to consider in the management?
A. The dose of thyroxine usually goes up by 20% in the third trimester
B. TSH should be tested only once in pregnancy
C. There is usually no change in the dose of thyroxine
D. Thyroxine crosses the placenta and can cause fetal goitre
E. The dose should be increased postpartum

Answer: C
For those women who are adequately replaced pre-pregnancy, there will be no change in the
thyroxine dose in pregnancy
23. A 28-year-old G2P1 who had an emergency caesarean section in her last pregnancy is
considered a VBAC. While counseling about the risks and benefits you must explain
about the risk of uterine rupture.
By what proportion does this risk increase?
A. 0.5 fold
B. 1-1.5 fold
C. 2-3 fold
D. 5 fold
E. >5 fold

Answer: B
Women should be informed of the two- to three-fold increased risk of uterine rupture and around
1.5-fold increased risk of caesarean section in induced and/or augmented labours compared with
spontaneous labours. Women should be informed that there is a higher risk of uterine rupture
with induction of labour with prostaglandins.

24. A woman suffers a massive postpartum haemorrhage during emergency caesarean


section. She is Rh-D negative and receives two units of Rh positive FFP and
cryoprecipitate.
What additional therapy should be given in this case?
A. 250 iu of Anti-D immunoglobulin
B. 500 iu of Anti-D immunoglobulin
C. Check Kleihauer and then decide dose
D. No Anti-D is required
E. 1500 iu Anti-D is given

Answer: D
No Anti-D prophylaxis is required if an Rh D negative woman receives Rh D positive FFP or
cryoprecipitate. For platelet transfusion usually 250 iu of Anti D is recommended for up to 4
pools of platelet.

25. In a woman undergoing exploratory surgery for staging of ovarian cancer, biopsies from
the retroperitoneal lymph nodes are taken.
What is the site of the primary lymphatic drainage from the ovaries?
A. External iliac LN
B. Superficial inguinal LN
C. Deep inguinal LN
D. Para-aortic LN
E. Hypogastric LN

Answer: D

26. A primigravida presents with abdominal pain at 28 weeks in an otherwise uneventful


pregnancy. She is diagnosed with a Group B bacteriuria on culture (>10+5 cfu/ml) and
candida on HVS. She is allergic to penicillin.
What is the recommended treatment?
A. 5 day course of metronidazole
B. 7 days course of clindamycin cream
C. Only clotrimazole is recommended
D. IV clindamycin in labour only
E. Clotrimazole and Clindamycin in labour

Answer: E

27. You have seen a couple in your clinic with primary subfertility for 3 years. The semen
analysis is normal but the male partner is found to have bilateral small varicoceles. All
other tests for the female is normal. What is the appropriate advice with regards to the
varicocele?
A. No surgery is indicated in this case
B. Surgery will improve chances of pregnancy
C. Surgery is indicated only when there is severe oligospermia
D. Surgery is indicated as the infertility is > 3 years
E. Review the varicoele in 6 months

Answer: D
Varicocele repair should be considered in case of a clinical varicocele, oligospermia, infertility
duration of ≥ 2 years, and otherwise unexplained infertility in the couple.

28. What is the commonest cause of postmenopausal bleeding?


A. Endometrial polyps
B. Endometrial hyperplasia
C. Endometrial cancer
D. Endometrial atrophy
E. Endometritis

Answer: D

29. A 62-year-old lady presents with a history of postmenopausal bleeding. She is not on
HRT.
What is her risk of endometrial cancer?
A. 1%
B. 5%
C. 10%
D. 20%
E. 30%

Answer: C

30. In women diagnosed with obstetric cholestasis soluble vitamin K is administered as 10


mg tablets.
What does this prevent in the fetus?
A. Neonatal haemolytic anaemia
B. Hyperbilirubinaemia
C. Kernicterus
D. Meconium passage
E. Neonatal bleeding

Answer: E
As vitamin K is fat soluble, women with fat malabsorption – especially biliary obstruction or
hepatic disease will have low levels of Vitamin K. Hence giving water soluble vitamin K is
recommended in women with liver disease. This will prevent maternal bleeding and fetal or
neonatal bleeding. The British National Formulary advises avoiding therapy in late
hyperbilirubinaemia and kernicterus in the baby.

31. A 20-year-old primigravida presents with a history of vomiting and nausea. She is
currently 12 weeks pregnant. In hyperemesis gravidarum, what should you prescribe for
the prevention of Wernicke’s encephalopathy?
A. Vitamin B1
B. Vitamin B2
C. Vitamin B6
D. Vitamin B12
E. Pantothenic acid

Answer: A

32. A woman who has asthma is 38 weeks pregnant. She is currently on 7.5 mg prednisolone
for control of her asthma.
Which medication would you use with caution in the management of her labour?
A. PGE2 for induction of labour should be used with caution
B. Hydrocortisone is not indicated in labour
C. Epidural analgesia should be used with caution
D. PGF2α for PPH should be used with caution
E. IV fluids should be used with caution

Answer: D

33. A 28-year-old pregnant lady who is known to have rheumatoid arthritis is seen in the
antenatal booking clinic.
What is important to consider about the course of the disease in pregnancy?
A. <10% improve significantly
B. 50% will deteriorate
C. 75% improve postpartum
D. 75% improve in pregnancy
E. Usually no change

Answer: D
Of those that improve 90% get a flare up postpartum.
34. A patient presents at the antenatal booking clinic. Her blood suggest B-thalassaemia trait.
What indices on her blood picture are you expecting to find?
A. High MCV, normal MCHC
B. Low MCV, low MCHC
C. High MCV, low MCHC
D. Low MCV, normal MCHC
E. Low MCV, high MCHC

Answer: D
Anaemia in women with thalassaemia is classic with low MCV and low MCH. The MCHC is
normal in contrast with iron deficiency anaemia, where all indices are low.

35. What is the most common neonatal condition in mothers who have SLE?
A. Congenital cardiac block
B. Renal abnormalities
C. Cutaneous neonatal lupus
D. Cardiac structural defects
E. Photosensitivity

Answer: C
The risk of neonatal cutaneous lupus is about 5%. The risk of congenital heart block is 2-5%.
The rash usually develops at 2 weeks of life and resolves by 6 months.

36. A woman who developed HELLP in her first pregnancy is extremely anxious about this
in her second pregnancy.
What is the risk of recurrence in her case?
A. 1%
B. 5%
C. 10%
D. 15%
E. 25%

Answer: B

37. A primigravida in her 35th week of an uncomplicated pregnancy develops itchy rashes
mainly on the abdomen and arms. Her liver function tests are normal, bile acid results are
unavailable.
What is the most likely diagnosis?
A. Obstetric cholestasis
B. Pemphigoid gestationis
C. Polymorphic eruption of pregnancy
D. Erythema multiforme
E. Prurigo of pregnancy

Answer: C
This rash is usually seen after the 35th week and more commonly in primigravidas. Usually
resolves postpartum and has no detrimental effect on the fetus.

38. A 65-year-old woman who had a hysterectomy and left oophorectomy for endometriosis
at age 43 presents with a 6 cm right sided multiloculated ovarian cyst on scan. There are
solid areas and some ascites is noted. No evidence of metastasis is seen. CA125 is 30.
What is the risk of malignancy index (RMI)?
A. 120
B. 90
C. 150
D. 270
E. 360

Answer: D
RMI: U (ultrasound score) x M (menopausal score is 3) x CA125
Ultrasound score: 1 for each of the points
- Solid areas
- Multilocular
- Ascites
- Evidence of metastasis
- Bilaterality

39. In a postmenopausal woman with a RMI of 230 what is the approximate risk of cancer?
A. 3%
B. 8%
C. 15%
D. 20%
E. 30%

Answer: E
Low risk (RMI <25, cancer risk is <3), Moderate risk (RMI 25-250, cancer risk is 30%), high
risk (RMI >250, cancer risk is 70%)

40. What is the risk of ovarian hyperstimulation in an IVF cycle?


A. 5%
B. 12%
C. 20%
D. 25%
E. 33%

Answer: E

41. A 33-year-old lady is admitted to the gynaecology ward with moderate abdominal pain
and distension. She had IVF treatment and the oocytes were retrieved about 10 days ago,
two embryos were transferred. She is sent for a scan. It shows ovaries measuring about
10 cm with 7 cm free fluid in pouch of Douglas.
What is the diagnosis?
A. Mild ovarian hyperstimulation
B. Moderate ovarian hyperstimulation
C. Severe ovarian hyperstimulation
D. Ovarian abscess
E. Critical ovarian hyperstimulation

Answer: C
Mild OHSS: Abdominal bloating, Mild abdominal pain, Ovarian size usually < 8 cm
Moderate OHSS: Moderate abdominal pain, nausea ± vomiting, ultrasound evidence of ascites,
ovarian size usually 8-12 cm
Severe OHSS: Clinical ascites (occasionally hydrothorax), Oliguria Haemoconcentration
haematocrit > 45%, Hypoproteinaemia, Ovarian size usually > 12 cm.
Critical OHSS: Tense ascites or large hydrothorax, Haematocrit > 55%, White cell count >
25,000/ml, Oligo/anuria, Thromboembolism, Acute respiratory distress syndrome.

42. A 24-year-old woman, is 26 weeks pregnant. She calls delivery suite complaining of
headache, joint pains, diarrhea and vomiting for 1 day and low grade fever since she
arrived from Sierra Leone 2 weeks back. She is brought to the delivery suite by
ambulance crew. With the given history what would be your provisional diagnosis?
A. Malaria
B. Listeria
C. Gastroenteritis
D. Ebola
E. Dengue

Answer: D
The main points in the care of women with suspected Ebola is
- Isolation in a separate room
- Minimal staff contact
- Appropriate protective gear
- Talk but don’t touch
- Resuscitation of patient with caution
- No CTG or fetal monitoring
- No operative delivery

43. A 35-year-old P2 with a booking BMI of 55 is 34 weeks pregnant. She has a proven
diagnosis of deep vein thrombosis and is on treatment with low molecular weight heparin
(LMWH).
Which test would you use for surveillance?
A. Platelet count
B. APTT
C. No surveillance required
D. Anti-Xa level
E. Repeat Doppler ultrasound
Answer: D
Anti Xa levels are recommended for extremes of weight (< 50 or > 90 kg) or where there is renal
impairment, but generally surveillance is not required.

44. A 28-year-old woman comes to you for advice regarding her inability to conceive despite
six months of consciously trying. She and her partner have no significant medical history
and stay together. Her BMI is 51kg/m2 and she gets her periods once every 45 days. To
expedite her conception you advise her that she needs to lose weight.
What is the most appropriate first line management for her?
A. Do nothing and reassure the patient a she is young and less than one year into her
marriage
B. Daily exercise appropriate to her endurance
C. Low calorie diet
D. Laparoscopic adjustable gastric banding
E. Appetite suppressants

Answer: D
Any person with BMI more than 50 kg/m2 should be offered bariatric surgery as a first line
management for her health as conventional methods of weight loss are not known to help in such
extreme cases.

45. A woman who has delivered 30 minutes ago has a retained placenta and has had about
600 mls of blood loss. Based on the recent NICE guidelines which drug would you
recommend after a retained placenta is diagnosed?
A. Intravenous oxytocin infusion
B. Umbilical vein oxytocin injection
C. Intramuscular ergometrine
D. Intramuscular carboprost
E. Umbilical artery oxytocin injection

Answer: A

46. A primigravida is in active labour. She has no progress at 7 cm and after 4 hours of
ARM. Syntocinon is commenced. How soon should there be a vaginal examination after
commencing syntocinon if the CTG is normal?
A. 1 hour
B. 2 hours
C. 3 hours
D. 4 hours
E. After 2 hours of contractions

Answer: B

47. In a woman who has been diagnosed with anogenital lichen sclerosus what is the first line
of management?
A. CO2 laser vaporization
B. Surgical excision
C. Topical tacrolimus
D. Topical ultrapotent steroid
E. Steroid with antifungal

Answer: A? D
Ultrapotent steroids such as Clobetasol Proprionate is the recommended first line of
management. Tacrolimus should be used in those resistant to steroids. Surgery and CO2 laser
vaporization are not recommended for the treatment of symptoms of lichen sclerosis. However,
these treatments have a cosmetic role in restoring function impaired by agglutination and
adhesions such as urinary retention or narrowing of the vaginal introitus that affects sexual
function or body image.

48. A young girl, 18 years of age was diagnosed with chlamydia infection. She and her
partner were treated. The current national chlamydia screening guidelines for women
under 25 recommend a retesting.
How soon should this be?
A. After 1 month
B. After 2 months
C. After 3 months
D. After 6 months
E. After 1 year

Answer: C

49. In a 65-year-old healthy woman with a vault prolapse what is the most effective
management?
A. Ring pessary
B. Pelvic floor exercises
C. Anterior and posterior repair with obliteration of enterocele
D. Intravaginal slingoplasty
E. Abdominal sacrocolpopexy

Answer: E
Conservative measures such as ring and shelf pessary and pelvic floor exercises are reserved for
women who do not wish or are not fit to have surgery. Slingoplasty again should be reserved for
women not suitable for surgery. They are not as effective as sacrocolpopexy which should be the
first line management for vault prolapse in fit and healthy women.

50. What is the main action of Mirabegron which is used to treat women with OAB
(Overactive bladder)?
A. β3 adrenoceptor agonist
B. β2 adrenoceptor antagonist
C. Muscarinic receptor agonist
D. Mixed α and β receptor agonist
E. Α 1 adrenoceptor agonist
Answer: A
It is a β3 adrenoceptor agonist, which activates β3 adrenoceptors causing the bladder to relax,
which helps it to fill and also to store urine. It is administered orally with the recommended dose
being 50 mg daily, and 25 mg if there is renal or hepatic impairment.

You might also like